14.07.2015 Views

Model Question Paper CET Physics - WIT Solapur

Model Question Paper CET Physics - WIT Solapur

Model Question Paper CET Physics - WIT Solapur

SHOW MORE
SHOW LESS

You also want an ePaper? Increase the reach of your titles

YUMPU automatically turns print PDFs into web optimized ePapers that Google loves.

18. The wavelength of photon and electron is λ ph and λ e and energy (E) of the two issame then :(1) the difference can be obtain if E is given(2) λe>λph(3) λph . λe(4) λph=λe19. A lift is moving with acceleration a in upward direction then the force applied bymass m on the floor of lift will be :(1) ma (2) m(g-a) (3) m(g+a) (4) mg20. Two cars of m 1 and m 2 mass are moving in the circular paths of r 1 and r 2 radius,their speed is such that they travels one cycle in the same time, the ratio of theirangular velocities is :(1) m 1 r 1 : m 2 r 2 (2) 1 : 1 (3) r 1 : r 2 (4) m 1 : m 221. A ring of mass M, radius r is moving with angular velocity w, if another twobodies each of mass m is placed on its diameter, the resultant angular velocity willbe :(1) w(M + 2m) (2) w(M – 2m) (3) wM (4) wMM (M + 2m) (m+m) (M+2m)22. The wavelength of 1 ke V photon 1.25 x 10 -9 m the frequency of Me V photonwill be:(1) 1.24 x 10 23 (2) 2.4 x 10 23 (3) 2.4 x 10 23 (4) 1.24 x 10 1523. Size of nucleusis of the order of :(1) 10 -13 cm (2) 10 -10 cm . (3) 10 -8 cm . (4) 10 -15 cm.24. If MI, angular acceleration and torque of body is I, ∝ and τ, it is revolving withω angular velocity then :(1) τ = α (2) M= 1 (3) τ= Iα (4) τ=IωIα25. In a uniform circular motion :(1) both acceleration and speed changes(2) both acceleration and speed are constant(3) both acceleration and velocity are constant(4) both acceleration and velocity changes26. Ratio of average kinetic evergies of H 2 and O 2 at a given temp. is :(1) 1 : 1 (2) 1 : 4 (3) 1 : 8 (4) 1 : 1627. To make the working of a machine, free of magnetism, the cover of this machinemust be of :(1) non magnetic substance(2) diamagnetic substance(3) paramagnetic substance(4) ferro magnetic substance


50. Relation between displacement x and time t is x = 2 – 5t + 6t 2 , the initial velocity willbe:(1)-3 m/sec. (2) 12 m/sec. (3) 2 m/sec. (4) – 5 m/sec.51. Focal length of a convex lens is 16 cm. it is dipped in water. The refractive indices ofthe substance of lens and water are 1.5 and 1.33 resp., now the focal length will be :(1) 64 cm. (2) 18 cm. (3) 24.24 cm. (4) 16 cm.52. In a half wave rectifier circuit, the input signal frequency is 50 Hz, the the outputfrequency will be :(1) 25 Hz (2) 50 Hz (3) 200 Hz (4) 100 Hz53. In the following circuit :B A PC D Q(1) the loop will be displaced along the length of wire(2) PQ unchanged(3) the loop will repell the wire(4) wire will attract the loop54. In a triode the ratio of small change in plate voltage and small changes in gridvoltage is, if plate current is constant :(1) DC plate resistance(2) mutual conductance(3) AC plate resistance(4) amplification factor55. Two particles accelerated with same voltage eneters in a uniform magnetic fieldperpendicularly, the radii of the circular paths is R 1 and R 2 , the charge on particles issame the ratio of m 1 is :2 m 2 2(1) R 2 (2) R 2 (3) R 1 (4) R 1R 1 R 1 R 2 R 256. Light Velocity in diamond is ( µ = 2.0)(1) 60 x 10 10 cm/sec.(2) 2 x 10 10 cm/sec.(3) 3 x 10 10 cm/sec.(4) 1.5 x 10 10 cm/sec.57. If Arsenic is dopped to silicon then its conductivity :(1) becomes zero(2) unchanged(3) increases(4) decreases


58. Two condensers of c and 2c capacity are connected in parallel and these are chargedupto v volt. If the battery is removed and dielectric medium of k constant is put betweenthe plates, then the potential at each condenser is :(1) v (2) 2+ k (3) 2v (4) 3vk + 2 3v k + 2 k+259. Equation of wave is y = 15 x 10 -2 sin (300t – 100x) where x in meter and t in sec. thewave velocity is :(1) 1.5 m/sec. (2) 3 m/sec. (3) 0.5 m/sec. (4) 1 m/sec.60. Escape velocity at the surface of earth is 11 km/sec., if radius of earth is doubled thenthe escape velocity will be :(1) 15.5 km/sec. (2) 5.5 km/sec. (3) 11 km/sec. (4) 22 km/sec.61. Kinetic energies of two bodies of 1 kg. and 4 kg. are same, the ratio of theirmomentum is :(1) 1 : 16 (2) 1 : 2 (3) √2 : 1 (4) 4 : 162. A body takes 5 minute to cool from 30 0 C to 50 0 C. How much time it will take tocool from 60 0 C to 30 0 C, if room temp. is 20 0 C :(1) 40 minute (2) 10 minute (3) 30 minute (4) 20 minute63. AC voltage is v = 200 sin 300t and if R = 10Ω and L = 800 mH, peak value of currentis :(1) 1.83 (2) 1.5 (3) 2.0 (4) .8364. Two charges + q and – q are placed at r distance from each other. If one of the chargeis stationary and other is rotated around, work done is one circle is :(1) kq 2 (2) kq (3) kq 2 (4) zeror 2 r r65. Peak value of AC current is 4√2, RMS current is :(1) 2√2 (2) 8 (3) 4 √2 (4) 466. A monoatomic gas is compressed to its 1/8 th volume adiabatically (r = 5/3), thepressure will be :(1) 32 times (2) 40 times (3) 8 times (4) 24 times3 567. A condenser is charged and then battery is removed, a dielectric plate is put betweenthe plates of condenser, then correct statement is :(1) Q constant V and U decreases(2) Q constant V increases U decreases(3) Q increases V decreases U increases(4) None68. The MI of a disc wrt its diameter is I, MI wrt. And axis passing through itscircumference and parallel to diameter is :


(1) 41 (2) 61 (3) 31 (4) 5169. Length of two wires is 0.5 m and distance between the wires is 1 m. If 1 amp.current is passed in the wires, force per unit length between the wires is :(1) 4 x 10 -7 (2) 2 x 10 -7 (3) 10 -7 (4) None70. Relation between internal energy U and absolute temp. T of an ideal gas as kinetictheory of gases, is :(1) U does not depends upon T(2) U ∝ T 2(3) U ∝ T(4) U ∝ √T71. Light wavelength in a glass is 6000Ǻ and refractive index is 1.5, the wavelengthof light is :(1) 12000 Ǻ (2) 4000 Ǻ (3) 9000 Ǻ (4) 6000 Ǻ72. Two sources of sound A & B placed near to each other produces 4 beats per second.If A is loaded with wax then 2 beats/sec. are produced. If the frequency of A is 256 Hz,The frequency of B will be :(1) 262 (2) 260 (3) 252 (4) 25073.Work done to rotate a dipole by a 90 0 angle, is :(1) – PE (2) – 2 PE (3) 2PE (4) PE74. Zener diode may be used as a :(1) rectifier (2) oscillator (3) amplifier (4) voltage regulator75. Wavelength of first line of Balamer series is 6561 Ǻ then the wavelength of secondline of Balmer series will be :(1) 3500 Ǻ (2) 4860 Ǻ (3) 6561 Ǻ (4) 2430 Ǻ


RAJASTHAN P.E.T.PHYSICS-19971. In Bohr’s theory the potential of an electron at a position is kr 2 , k isconstant, then the quantized energy of the electron in n th orbit : 21 12(1) nh k (2) nh k(3) nh m (4) nh m 2m m k k2. To reduce the de-Broglies wave length of an electron from 100 pm to 50 pm,the required increase in energy is :(1) 150 eV (2) 300 eV (3) 450 eV (4) 600 eV3. The angular width of fringes in Young’s bislit experiment is 0.20 0 with thewavelength 5890 Å. If the whole apparatus is dipped in water, the angularwidth will be :(1) 0.30 0 (2) 0.22 0 (3) 0.15 0 (4) 0.11 04. Resistance of a 10 m. long wire of potentio meter is 1 ΩΩ Ω/m. A cell of 2.2 volt emf.and HRB is connected in series with the wire. How much resistance must be appliedto get 2.2 mv gradient :mt(1) 1000 Ω (2) 990 Ω (3) 810 Ω (4) 790 Ω5. Four charges are placed on corners of a square, having side of 5 cm. , if q isone coulomb then electric field intensity at the centre will be :q -2q-q + 2q(1) 1.02x10 7 N/c upwards(2) 2.04x10 7 N/c upwards(3) 2.04x10 7 N/c down(4) 1.02x10 7 N/c down6. Capacitance of a capacitor made by a thin metal foil is 2 µµµ F. If the foil isfilded with paper of thickness 0.15 mm. and dielectric constant of paper is2.5, width of paper is 40 mm. then length of foil will be :(1) 33.9 mm. (2) 13.4 mm. (3) 1.33 mm (4) 0.34 mm.


7. An electron and an αα- particle are accelerated with v volt voltage. If themasses are m e and m ααthen the ratio of momentum is :(1) 2m e (2) m e (3) m e (4) m em α 2m α m α m α8. Ultra sonic sound can be observed by :(1) Telephone (2) Hebb method (3) Quincke tube (4) Kundit tube9. Which two of the given transverse waves will give stationary wave when getsuper imposed :z 1 = a cos (kx - ωt) ….. Az 2 = a cos (kx _cot ) …..Bz 3 = a cox (ky - ωt) …..C(1) A and B (2) A and C (3) B and C (4) any two10. For what value of R the net resistance of the circuit will be 18 ohms :( 1) 24 Ω R(2) 16 Ω 10(3) 10 Ω 10(4) 8 Ω 1010 10 1011. For a medium refractive indices for violet, red and yellow are 1.62, 1.52 and1.55 resp. then dispersive power of medium will be :(1) 0.02 (2) 0.18 (3) 0.22 (4) 0.6512. The temperature at which the rms speed of hydrogen molecule is equal toescape velocity on earth surface will be :(1) 10059 K (2) 8270 K (3) 5030 K (4) 1060 K13. The temperature of a liquid drops from 365 K to 361 K in 2 minutes. Findthe time during which temperature of the liquid drops from 344 K to 342 K.Room temp. is 294 K.(1) 60 sec. (2) 66 sec. (3) 72 sec. (4) 84 sec.14. Venturimeter is used to measure :(1) surface teusion of liquid(2) rate of flow of liquid(3) density of liquid(4) pressure of liquid


15. A rod is fixed between two points at 20 0 C, coefficient of linear expansion ofmaterial of rod is 1.1 x 10 -5 / 0 C and Young’s modulus is 1.2 x 10 11 N/m. Findthe force developed in the rod it temp. of rod becomes 10 0 C :(1) 1.1 x 16 6 N/m 2(2) 1.1 x 10 15 N/m 2(3) 1.2 x 10 7 N/m 2(4) 1.32 x 10 8 N/m 216. If an air bubble of radius 1 mm. moves up with uniform velocity of 0.109cm/s. in a liquid column of density 14.7 x 10 3 kg./m 3 . If g = 10 m/sec. 2 thencoefficient of viscosity will be :(1) 10.0 m=sec. 2(2) 9.78 m-sec. -2(3) 9.62 m-sec. -2(4) 9.86 m-sec. -217. A rocket launched with 10 km/sec. velocity radius of earth is R, then themaximum height attained by it will be :(1) 5 R (2) 4 R (3) 3 R (4) 2 R18. A block of 2 kg. mass and body of 1 kg. mass are connected with the two endsof a string. The string is passing through a pulley. The block is put on ahorizontal table and the body is hanging. The table is friction less thenacceleration and force of tension are:(1) 4.38 ms -2 , 9.86 N(2) 4.38 ms -2 , 6.54 N(3) 3.27 ms -2 , 6.54 N(4) 3.27 ms -2 , 9.86 N19. A mass m performs oscillations of period T, when hanged by spring of forceconstant k, If spring is cut in two parts and arranged in parallel, If samemass is oscillated by them, new time period will be :(1) T (2) 2 T (3) T__ (4) T2 √220. In a triode amplifier µµ= 70, gm= 1600 µµmho and R L = 0.1 MΩΩ Ω. If input of 1v(rms) is given then power gained in load will be:(1) 4.87 mω (2) 23.7 mω (3) 2.37 mω (4) 48.7 mω21. Moment of inertia a rectangular thin plate having mass m, length ιι ι, width b,about an axis passing through its centre and perpendicular to the plane is :(1) Mι 2 (2) Mb 2 (3) M(ι 2 +b 2 ) (4) M(ι 2 +b 2 )12 12 3 1222. In a triode circuit for a given plate voltage, plate current will be maximumwhen:


(1) V g Positive and V p negative(2) V g and V p both positive(3) V g = 0 and V p positive(4) V g negative and V p positive23. In p-n function avalanche current flows in circuit when be maximum when :(1) excess (2) zero (3) reverse (4) forward24. Half life of a radioactive element is 10 days. The time during which quantityremains 1/10 of initial mass will be :(1) 16 days (2) 33 days (3) 50 days (4) 100 days25. Resistance of semiconductor at OK is :(1) small (2) large (3) infinity (4) zero26. αα α- particle of 400 KeV energy are bombarded on nucleus of 82 pb. Inscattering of αα α-particles, its minimum distance from nucleus will be :(1) 0.59 pm (2) 5.9 pm (3) 0.59 nm (4) 0.59 Å27. If the uncertainty in the position of an electron is 2Å then the uncertainty inthe energy is (about) :(1) 94 eV (2) 9.0 eV (3) 1.0 eV (4) 0.1 eV28. Wrong statement is :(1) Nuclear force is produced by the exchange of poins(2) Nuclear force increases with increase in no. of nucleous(3) Range of nuclear forces is very small(4) Nuclear forces are strongest29. The inductance required to connect bulb in series of 1 :(1) 1.62 mH (2) 16.2 mH (3) 2.42 mH (4) 1.27 mH30. A block follows the path as shown in the figure from height h. If radius ofcircular path is r, then relation holds good to complete full circle ish…………………………………..r


(1) h ≥ 5r2(2) h > 5r2(3) h ∝ 5r2(4) h = 5r231. A hollow sphere has 6.4 m radius minimum velocity required by a cyclist atbottom to complete circle will be :(1) 16 ms -1(2) 12.4 ms -1(3)10.2 ms -1(4) 8 ms -132. A block is lying on an inclined plane which makes 60 0 with the horizontal. Ifcoefficient of friction between block and plane is 0.25 and g = 10 ms -2 . Theacceleration of block when it moves along the plane will be :(1) 86 m/sec. (2) 99 m/sec. (3) 124 m/sec. (4) 172 m/sec.33. A charge moves in a circle perpendicular to a magnetic field. The time periodof revolution is independent of :(1) velocity (2) mass (3) charge (4) magnetic field34. A coil of 40 ΩΩresistance has 100 turns and radius 6 mm. is connected toammeter of 160 ohm resistance. Coil is placed perpendicular to the magneticfield. When coil is taken out of the field 32 µµ µc charge flows through it. Theintensity of magnetic field will be :(1) 0.566 T (2) 0.655 T (3) 5.66 T (4) 6.55 T35. A choke coil of 0.1 H inductance and 12 ΩΩresistance. If it is connected to 60Hz alternating current source the power factor will be:(1) 0.24 (2) 0.28 (3) 0.30 (4) 0.3236. For a gas C v = 4.96 cal/mole-K, when 2 mole gas is heated from 340K to 342K, increase in internal energy is :(1) 9.92 cal. (2) 13.90 cal. (3) 19.84 cal. (4) 27.80 cal.37. Luminous intensity for a bulb of 40 watt at 110 V is 11.01 lumen/watt. Thedistance at which intensity of illumination is 5 lumen/mt 2 will be:(1) 44.04 m (2) 18.78 m (3) 9.39 m (4) 4.40 m38. A 2m long rod of radius 1 cm. which is fixed from one end is given a twist of0.8 radians. The shear strain developed will be :(1) 0.016 (2) .008 (3) .004 (4) .002


39. A plane mirror makes an angle 300 with horizontal. It a vertical ray strikesthe mirror, find the angle between mirror and reflected ray :(1) 90 0 (2) 60 0 (3) 45 0 (4) 30 040. A rod of length ιιand radius r is joined to a rod of length ιι ι/2 and radius r/2 ofsame material. The free end of small rod is fixed to a rigid base and the freeend of larger rod is given a twist of θθ θ, the twist angle at the joint will be :(1) 8θ (2) 5θ (3) θ (4) θ9 6 2 441. At NTP one mole of diatomic gas is compressed adiabatically to half of itsvolume (r= 1.41). The work done on gas will be :(1) 2025 J (2) 1815 J (3) 1610 J (4) 1280 J42. An achromatic combination is made with a lens of f focal length and Wdispersive power with a lens having dispersive power of 2ωω ω. The work doneon gas will be :(1) - 2f (2) - f (3) f (4) 2f2 243. An electron and proton lying 10 cm. apart. The ratio of electrostatic forceand gravitational force between them will be:(1) 10 42 (2) 10 39 (3) 10 27 (4) 10 1944. Two wires A and B of same material have radius 2r r. If resistance of B willbe :(1) 17 Ω (2) 68 Ω (3) 272 Ω (4) 544 Ω45. A charged plate has charge density of 2 x 10 -6 c/m 2 . The initial distance of anelectron which is moving towards plate, can not strike the plate, if it is havingenergy of 100 eV :(1) 3.51 cm. (2) 1.77 cm. (3) 3.51 mm. (4) 1.77 mm.46. A sphere of radius 1 cm. has potential of 8000 V then the energy density nearits surface will be :(1) 2.83 Jm -3 (2) 8 x 10 3 Jm -3 (3) 32 Jm -3 (4) 64 x 105 Jm -347. A proton of 200 Me V energy enters the magnetic field of 5 T. If direction offield if from south to north and motion is upwards the force acting on it willbe :(1) 1.6 x 10 -6 N (2) 1.6 x 10 -10 N (3) 0 (4) 3.2x10 -8 N48. If V AB = uv in given figure then resistance X will be :10Ω 5 V(1) 20(2) 15


(3) 10 A 2 V X B(4) 549. A charged water drop whose radius is 0.1 µµm is equilibrium in an electricfield. If charge on it is equal to charge of an electron will be ( g= 10 ms -2 ) :(1) 1610 NC -1 (2) 262 NC -1 (3) 26.2 NC -1 (4) 1.61 NC -150. The charge on 500 ml. water due to protons will be :(1) 1.67x10 23 (2) 1.67x10 26 (3) 6.0x10 27 (4) 6x10 2351. A piece of cloud having area 25x10 6 m 2 and electric potential of 10 5 volt. Ifthe height of cloud is 0.75 km. then the energy density of electric fieldbetween earth and cloud will be :(1) 1475 J (2) 1225 J (3) 750 J (4) 250 J52. 1 Farad in esu is :(1) 1 x 10 -6 (2) 9 x 10 11 (3) 3 x 10 10 (4) 1 x 10 -113 953. Electric potential is given by : V = 6x – 8xy 2 – 8y + 6yz – 4z 2 then the electricforce acting on 2 coulomb point charge placed on origin will be :(1) 2 N (2) 6 N (3) 8 N (4) 20 N54. The wavelength of K ααlines given by Molybdenum (At No. 42) is 0.7078 Åthen wavelength of K ααfor zinc ( At no. 30) will be :(1) 0.3541 Å (2) 1.3873 Å (3) 0.9425 Å (4) 1.2547 Å55. A plane wave front of 7000 Å fallson an aperture. The area of half periodzone of the diffraction pattern on screen 1 meter away from the aperture willbe :(1) 28x10 -7 m 2 (2) 44x10 -7 m 2 (3) 22x10 -7 m 2 (4) 14x10 -7 m 256. In Young’s double slit experiment 62 fringes are seen in visible region forsodium light of wavelength 5893 Å. If violet light of wave length 4358 Å isused in place of sodium light then number of fringes seen will be :(1) 84 (2) 74 (3) 64 (4) 5457. Average wavelength of light emitted by a 100 watt bulb is 5000 Å. The no. ofemitted photons per second :(1) 5x10 17 (2) 2.5x10 22 (3) 3x10 23 (4) 2.5x10 1958. To see first 20 lines of Balmer series distinctly minimum resolving power ofinstrument should be :(1) 1040 (2) 983 (3) 920 (4) 878


59. If diffraction pattern of an electron from a crystal is same as diffractionpattern of X-rays of wavelength 0.61 Å. The energy of electron beam is :(1) 0.4 keV (2) 1 ke V (3) 4 ke V (4) 50 ke V60. Two parallel pillars are 11 km. away from an observer. The minimumdistance between the pillars so that they can be seen separately will be :(1) 183 m. (2) 915 m (3) 20.8 m. (4) 3.2 m61. The focal length of objective and eyepiece of a telescope are 100 cm. and 5cm. Final image is formed at least distance of distinct vision. Themagnification of telescope is :(1) 20 (2) 24 (3) 30 (4) 3662. A planet is revolving around the sun. The average distance of the plant fromthe sum is 1.588 times than that of earth from sun . The time period of theplanet is :(1) 2 yrs. (2) 1.89 yrs. (3) 1.59 yrs. (4) 1.25 yrs.63. Time period of a brass pendulum is 1 sec. at 20 0 C. Linear expansion coeff is1.93 x 10 -5 ( 0 C) -1 . At 30 0 C temp. how much the clock will be back in a week(1) 504s (2) 224s (3) 56s (4) 8s64. Mass and radius of the earth is M and R. Wrok done to bring a 1 kg. massfrom surface to the infinity is : _____ _____(1) GM (2) GM (3) √GM (4) √2GM2R R 2R R65. In the following reaction what are the values of A,B,C,D and E:92U 238 →→B Th AB →→D Pa CE →→92 U 234(1) A = 234, B = 90, C = 234, D = 93, E = α(2) A= 238, B = 93, C = 234, D = 91, E = β(3) A = 234, B = 90, C = 238, D = 94, E = α(4) A = 234, B = 90, C = 234, D = 91, E = β66. A bomb of 12 kg. divides in two parts ratio of masses is 1 : 3. If kinetic energyof smaller part is 216 J, then the momentum of bigger part in kg-m/sec. is :(1) 108 (2) 72 (3) 36 (4) Data is incomplete67. Weight of 1 kg. becomes 1/6 on moon, if radius of moon is 1.768 x 10 6 . Massof moon will be :(1) 7.65 x 10 22 kg. (2) 7.56 x 10 26 kg. (3) 5.98 x 10 24 kg. (4) 1.99x10 30 kg.68. Due to some force F 1 a body oscillates with period 4/5 sec. and due to otherforce F 2 oscillates with 3/5 sec. If both forces act simultaneously new periodwill be :


(1) 0.36 sec. (2) 0.48 sec. (3) 0.72 sec. (4) 0.64 sec.69. A wave is given by y = 3 sin 20 1 - x where y in cm.0.04 0.01frequency of wave and maximum acceleration will be :(1) 25 Hz, 7.5 x 10 4 cm.-sec .-2(2) 25 Hz, 4.7 x 10 4 cm.-sec. -2(3) 50 Hz, 7.5 x 10 3 cm.-sec. -2(4)100 Hz, 4.7 x 10 3 cm.-sec. -270.Two forces of 5 and 10 dynes resp. are acting on a particle, the resultant forcenever can be :(1) 8 dyne (2) 5 dyne (3) 12 dyane (4) 4 dyne71.A boggy of uniformly moving train is suddenly detached from train and stopsafter covering some distance. The distance covered by the boggy and distancecovered by the train in the same time has relation :(1) no definite ratio(2) first will be ¼ of second(3) first will be ½ of second(4) both will be equal72.πππ mesons can be :(5) π + , π-, π 0(6) π + and π -(7) π + , π 0(8) π - and π 073.In helium nucleus there are :(9) 2 positron, 2 neutrons(10) 2 protons, 2 neutrons(11) 2 protons, 2 neutrons, 2 electrons(12) 2 protons, 2 electrons74.Equivalent energy of 1 amu is :(13) 9.31 MeV(14) 931 KeV(15) 93.1 MeV(16) 931 Mev75.Density of nucleus is related to mass no. by :(1) ρ ∝ 1_ (2) ρ ∝ √A (3) ρ ∝ A (4) ρ = constantA76.The particles emitted by radio active decay are deflected by magnetic field.The particles will be :


(17) electron and α-particle(18) electron, proton and neutron(19) electron, proton and α(20) proton and α77.At 0 0 K Fermi level for metals :(21) depends on metal(22) lies between empty levels(23) lies between filled levels(24) separate empty and filled levels78.If quantity of a radioactive element remains 1 of initial one in 30 yrs. Half lifeof this element will be : 16(1) 24 yrs. (2) 18 yrs (3) 7.5 yrs (4) 1.9 yrs.79. The second’s hand of a watch has length 6 cm. speed of end point andmagnitude of difference of velocities at two perpendicular positions will be :(25) 6.2 and 8.8 mm-sec. -1(26) 8.88 and 6.28 mm-sec. -1(27) 8.88 and 4.44 mm-sec. -1(28) 6.28 and zero mm-sec. -180.A meter scale is standing straight vertically on a table. The velocity of upperend, when it strikes the table. When lower end is fixed will be :(1) 1.7 ms -1 (2) 5.4 ms -1 (3) 8.7 ms -1 (4) 10.9 ms -181.Fundamental frequency of an open pipe is :(1) 15 Hz (2) 20 Hz (3) 30 Hz (4) 10 Hz82.The cause of Fraunhoffer’s lines is :(1) diffraction (2) interference (3) emission (40 obsorption83.Wavelength of third line of Balmer series for H ion is 108.5 mm. The bindingenergy of electron in the ion is :(1) 122.4 eV (2) 54.4 eV (3) 13.6 eV (4) 3.4 eV84.Wavelengths of extreme lines of Paschen series for hydrogen is :(29) 2.27 µm and 7.43 µm(30) 1.45 µm and 4.04 µm(31) 0.818 µm and 1.89 µm(32) 0.365 µm and 0.656 µm85.An ionic atom is equivalent to hydrogen atom has wavelength equal to ¼ ofthe wavelengths of hydrogen lines. The ion will be :(1) He + (2) Li ++ (3) Ne ++ (4) Na+ 10


86.An observer standing at station observes frequency 219 when a trainapproaches and 184 when train goes away from him. If velocity of sound in air is340 m/sec., then velocity of train and actual frequency of whistle will be :(33) 32.5 ms-1, 205 Hz(34) 29.5 ms-1, 205 Hz(35) 25.5 ms-1, 200 Hz(36) 29.5 ms-1, 200 Hz87.The kinetic energies of two bodies of 4 kg. and 16 kg. mass is same, the ratioof their momentum is :(1) 4 : 1 (2) 1 : 2 (3) 2 : 1 (4) 1 : 488.Wave length of light emitted by a star is shifting towards the red end, then thestar:(37) moving towards earth(38) moving far from earth(39) nothing can be said(40) is stationery89.In the following diagram a rectangular coil is placed in 0.25 T uniformmagnetic field, the area is 96 x 10 -4 M 2 and no. of turns is 50, 2 amp current isflowing then the torque is :NS(1) 0.24 N-m (2) 0.96 N-m (3) 0.36 N-m (4) 0.48 N-m90.Plate resistances of two triode values is 4 kΩΩΩ and 8 kΩΩΩ and amplificationcoeff. If 40. If used as amplifiers with these load resistances then the ratio ofvoltage gains is :(1) 10 (2) ¾ (3) 16/9 (4) 4/391.Two particles of same mass are moving in the circular paths r 1 and r 2 radius,the ratio of their centripetal forces is :(1) √r 2 : √ r 1 (2) √r 1 : √r 2 (3) r 1 : r 2 (4) r 2 : r 192.In an AC circuit R = 100 ΩΩ Ω, L = 800 mH and E = 200 sin 300t then the peakvalue current is :(1) 1.17 A (2) 0.83 A (3) 0.59 A (4) 1.70 A


93.Length of wire of potentio meter is 100 cm. and resistance is 0.005 ΩΩ Ω/cm. Abattery of 2.0 volt emf and 1.5 ΩΩinternal resistance is connected at the ends ofthe wire then the value of potential gradient is :(1) 4 x 10 -4 v/m (2) 0.005 v/m (3) 0.05 v/m (4) 0.5 v/m94.RMS velocity of a gas molecules is 300 M/s at a given temperature. RMSvelocity of a gas, of which molecular weight is double and temp. is half of that ofthe first gas, is :(1) 150 m/sec. (2) 300 m/sec. (3) 300 √2 m/sec. (4) 600 m/sec.95.Two cars are moving on two perpendicular roads towards a crossing withuniform speeds of 72 km/hr. and 36 km/hr. If first car blows horn of 280 Hzfrequency, then the frequency heard by the driver of second car when linejoining the cars 450 angle with the roads will be :(1) 280 Hz (2) 289 Hz (3) 298 Hz (4) 321 Hz96.A disc of 1/3 m radius is hanged by a point on circumference by horizontalrail. Period of oscillation is 1.42 sec. value of g by this experiment will be :(1) 10.0 m-sec -2 (2) 9.78 m-sec. -2 (3) 9.62 m-sec. -2 (4) 9.86 m-sec -297. Two masses of 5 kg. each falling from height 10 m. , by which 2 kg. water isstirred. The rise is temp. of water will be :(1) 0.12 0 (2) 0.32 0 (3) 1.2 0 (4) 2.6 098.A circular road of 1000 m radius has banking angle 45 0 , the maximum safespeed of a car having 2000 kg. mass will be, if the coefficient of friction betweentyre and road is 0.5.(1) 86/ m/sec. (2) 99 m/sec. (3) 124 m/sec. (4) 172 m/sec.


Chemistry


www.JbigDeaL.comRAJASTHAN P.E.T.CHEMISTRY – 19971. The hybridization state of C atom in butendioic acid is :(1) sp 2 (2) sp 3 (3) both two (4) sp2. Which of the following is not a isomer of pentane :(1) n-pentane(2) 2, 2-dimethy 1 propane(3) 2, 3-dimethy 1 butane(4) 2-methy 1 butane3. The oxidation number of C atom in Ch 2 CI 2 and CCI 4 are respectively :(1) -2 and – 4 (2) 0 and – 4 (3) 0 and 4 (4) 2 and 44. Which of the following dissolves in lonic solvents :(1) C 6 H 5 (2) CH 3 OH (3) CCI 4 (4) C 5 H 125. The conjugate acid of HS is :(1) S -2 (2) H 2 S 2 (3) both two (4) none6. Phenolphthalein of pH range [8-10] is used in which of the following type oftitration as a suitable indicator :(1) NH 4 OH and HCI(2) NH 4 OH and HCOOH(3) NH 4 OH and C 2 H 4 O 2(4) NaOH and C 2 O 4 H 27. Which of the following is iron are :(1) Malachite (2) Hernatite (3) Siderite (4) Limonite8. The molar concentration of chloride ions in the resulting solution of 300ml.of 3.0 M NaCI and 200 ml. of 4.0 M BaCl 2 will be :(1) 1.7 M (2) 1.8 M (3) 5.0 M (4) 3.5 M9. Which of the following has least bond energy :-2-+(1) N 2 (2) N 2 (3) N 2 (4) N 210. Which of the following species has highest bond energy :-2+-(1) O 2 (2) O 2 (3) O 2 (4) O 211. Which of the following compound is not aromatic :(1) 1, 3-cyclobutene(2) pyridine(3) furane(4) thiophene


www.JbigDeaL.com12. Which of the following compound is used as refrigerant :(1) CCI 2 F 2(2) CCI 4(3) CF 4(4) Acetone13. Which of the following is weak acid :(1) C 6 H 6 (2) CH 3 -C≡CH (3) CH 2 =CH 2 (4) CH 3 -C≡C-CH 314. L.P.G. mainly consist of the following :(1) Methane (2) Hydrogen (3) Acetylene (4) Butane15. The solubility product of CaCo 3 is 5 x 10 -9 . The solubility will be :(1) 2.5 x 10 -5 (2) 7 x 10 -5 (3) 2.5 x 10 -4 (4) 2.2 x 10 -916. The outer electronic configuration of alkali earth metals is :(1) nd 10 (2) ns 1 (3) np 6 (4) ns 217. The nature of 2, 4, 6-trinitrophenol is :(1) Neutral (2) Basic (3) Acidic (4) Weak basic18. Which of the following group is sharp ortho and para directive :(1) –C 6 H 5 (2)-OH (3) –CH 3 (4) –CI19. By which of the following process hydrocarbons are found from petroleum :(1) combustion(2) fractional distillation(3) addition(4) all above20. A sample of petroleum contains 30% n-heptane, 10% 2-methyl hexane and60% 2, 2, 4-trimethyl pentane, the octane no. of this sample will be :(1) 30% (2) 60% (3) 10% (4) 70%21. In which of the following halogens p-electrons does not take part inresonance :(1) CH 2 =CH-CH 2 Cl (2) BrC 6 H 5(3) C 6 H 5 Cl (4) CH 2 =CHCl22. Which of the following statement is false :(1) 40% solution HCHO is known as formalin(2) HCHO is least reactive in its homologous series(3) The B.P. of isovarelaldehyde is less than n-varelaldehyde(4) The boiling point of ketones are higher than that of aldehydes23. If n + ιι= 8 then the expected no. of orbitals will be :(1) 4 (2) 9 (3) 16 (4) 25


www.JbigDeaL.comAlc. KOH 2Cl 2 Ca(OH) 224. A B C here the compound C will be :(1) Lewsite (2) Westron (3) Acetylene tetra chloride (4) Both 2 and 325. Which of the following is least hydrolysed :(1) BeCl 2 (2) MgCl 2 (3) CaCl 2 (3) BaCl 226. The laughing gas is :(1) N 2 O 4 (2) NO (3) N 2 O (4) N 2 O 527. The hydrogen ion concentration of a solution is 3.98 x 10 -6 mole per liter. ThepH value of this solution will be :(1) 6.0 (2) 5.8 (3) 5.4 (4) 5.928. The reaction of sodium acetate and sodalime gives :(1) Butane (2) Ethane (3) Methane (4) Propane29. Which of the following acids does not contain – COOH group :(1) Carbamic acid (2) Barbituric acid(3) Lactic acid (4) succinnic acid30. Which of the following compound of xenone does not exists :(1) XeF 6 (2) XeF 4 (4) XeF 5 (4) XeF 231. FeSO 4 , 7H 2 O is :(1) Mohr’s salt (2) Blue vitriol (3) Green vitriol (4) White vitriol32. The solution of BiCl3 in dil. HCI when diluted with water white precipitate isformed which is :(1) Bismith oxychloride (2) Bismith oxide(3) Bismith hydroxide (3) none of these33. The strongest acid is :(1) acetic acid(2) trichloroacetic acid(3) dichloracetic acid(4) monochloroacetic acid34. The false statement regarding alkane is :(1) This does not perform polymerization reaction(2) This does not gives elimination reaction(3) It does not disappear the colour of dilute KMnO 4 solution(4) It does not decolourise bromine water35. Which of the following is strongest base :(1) C 6 H 5 NH 2 (2) CH 3 NH 2


www.JbigDeaL.com(3) NH 3 (4) CH 3 CONH 236. Which of the following aromatic compound gives sulphonation reaction veryeasily :(1) Chlorobenzene (2) Nitrobenzene (3) Toluene (4) benzene37. The geometry of I3- is :(1) Triangular (2) Linear (3) Tetrahedral (4) T-shape38. The half life of a radio active element is 140 days. 1 gm. of this element after560 days will become :(1) 1 gm (2) 1 gm (3) 1gm. (4) 1 gm.16 4 8 239. The volume concentration of hydrogen peroxide 6.8% concentration will be :(1) 5 (2) 11.2 (3) 22.4 (4) 2040. Which of the following on combustion give maximum energy :(1) Ethane (2) Propane (3) Methane (4) ButaneAnhy. AICI 341. C6H6 + CH3CI C6H5CH3 + HCI The name of above reaction is :(1) Gattermann (2) Reimer-tiemann(3) Friedel-Craft (4) Cannizaro42. The oxidation state of Cr in K 2 Cr 2 O 7 is :(1) + 4 (2) + 3 (3) + 6 (4) + 543. The natural rubber is the polymer of :(1) 1, 3- butadiene (2) polyamide (3) isoprene (4) none of these44. Nylone-66 is a :(1) polyester (2) polyamide (3) polyacrylate (4) none of these45. 2NO(g) + CI 2 (g) →→2 NOCI The equilibrium constant for this reaction is :←←(1) K c = [NOCI] 2 (2) K c = [NOCI] 2 ___[NO] 2 [CI 2 } 2 [2NO] 2 [CI 2 ](3) K c = [NOCI] 2 __ (4) K c = [2NOCI]____[NO] 2 [CI 2 ][2NO][CI]A46. C 6 H 6 + CO + HCI C 6 H 5 CHO + HCI here A is :(1) anhydrans ZnO (2) V 2 O 5 /450 0 C(3) anhydrous AICO 3 (4) solid KOH


www.JbigDeaL.com47. The values of for HCN and CH 3 COOH are 7.2 x 10 -10 and 1.75 x 10 -5 (at 25 0C) respectively. The strongest acid amongst them is :(1) CH 3 COOH (2) HCN (3) both (4) none of these48. In which of the following carbon atom (asterisk) is asymmetric :(1) CH 3 CH 2 CH (CH 3 ) CH 2 OH(2) CH 3 CH 2 CH (CH 3 ) CHOH(3) CH 3 CH 2 CH 2 CH 2 CH 2 OH(4) CH 3 CH 2 CH (CH 3 )CH 2 OH49. Benzene reacts with CH 3 COCI in presence of Lewis acid AICI 3 to form :(1) Acetophenone (2) Toluene (3) Benzyl Chloride (4) Chlorobenzene50. Which of the following is reducing agent :(1) H 2 S (2) HNO 3 (3) H 2 O (4) K 2 Cr 2 O 751. In which of the following alkyl chloride the possibility of SN 1 reactionmechanism is maximum :(1) (CH 3 ) 2 CHCI (2) (CH 3 ) 3 C-CI (3) CH 3 CI (4) CH 3 CH 2 CI52. The energy produced realated to mass decay of 0.02 amu is :(1) 28.2 MeV (2) 931 MeV (3) 18.62 MeV (4) none of these53. The mole of hydrogen ion in 50 ml. of 0.1 M HCI solution will be :(1) 5 x 10 2 (2) 5 x 10 -3 (3) 5 x 10 3 (4) 5 x 10 -254. Petroleum is mainly consist of :(1) Aliphatic alcohol(2) Aromatic hydrocarbon(3) Alipnetic hydrocarbon(4) None of these∆∆55. C 6 H 6 OCH 3 + HI …….. + ……….. The products in the above reactionwill be :(1) C 6 H 5 I+CH 3 OH (2) C 6 H 5 CH 3 +HOI(3) C 6 H 5 OH+CH 3 I (4) C 6 H 6 +CH 3 OI56 F3 is :(1) Bronsted base (2) Lewis base (3) Lewis acid (4) Bronsted acid57. Which of the following compound gives violet colour with FeCI 3 solution:(1) Benzaldehyde (2) Aniline (3) Nitrobenzene (4) Phenol58. Hypo solution forms which of the following complex compound with AgCI :(1) Na 5 [Ag(S 2 O 3 ) 3 ] (2) Na 3 [Ag(S 2 O 3 ) 2 ]


www.JbigDeaL.com(3) Na 2 {Ag(S 2 O 3 ) 2 ] (4) Na 3 [Ag(S 2 O 3 ) 3 ]59. Molecular oxygen is :(1) ferro magnetic (2) diamagnetic (3) para magnetic (4) non magnetic60. Bonds in acetylene are :(1) 2π bonds (2) one π bond (3) 3π bonds (4) none of these61. The false statement for Griynaed reagent is :(1) It gives tertiary alcohol with acetamide(2) It gives tertiary alcohol with acetone(3) It gives secondary alcohol with acetaldehyde(4) It gives primary alcohol with formaldehyde62. Which of the following alkane exists is liquid state at normal temperature :(1) C 20 H 42 (2) C 3 H 8 (3) C 8 H 18 (4) CH 463. The solubility of AgCI at 25 0 C will be maximum in :(1) Potassium chloride solution(2) AgNO 3 solution(3) Water(4) All above64. The weight of a benzene molecule is :(1) 78 gm. (2) 7.8 gm. (3) 13 x 10 -23 (4) none of these65. CuFeS 2 is :(1) iorn pyrites (2) malachite (3) chalcosite (4) chalcopyrites66. Primary halides follow the following reaction mechanism :(1) SN 1 (2) SN 2 (3) both (4) none of these67. C and Si belong to the same group of periodic table, CO 2 is a gas and SiO 2 is a :(1) liquid (2) gas (3) solid (4) none of these68. H 2 S is a gas while H 2 O is a liquid because :(1) there is association due to hydrogen bonding(2) bond energy of OH high(3) the ionization potential of oxygen is high(4) the electro negativity of oxygen is high69. “The negative part of the molecule adding to the double bond goes to thatunsaturated asymmetric carbon atom which is linked to the least number ofhydrogen atoms.” This statement is related to :(1) Markownikoff’s law(2) Peroxide effect(3) Bayer’s law of distortion


www.JbigDeaL.com(4) none of these70. The conjugate base of NH3 is :(1) N 2 H 4 (2) NH 2-(3) NH 4+(4) NH 2+71. (a) N 2 and (b) C 2 H 2 . The nos. of ππand σσbond in the molecules are respectively :(1) (a) 2,2 (b) 2,2 (2) (a) 1,2 (b) 2,1(3) (a) 2,1 (b) 2,3 (4) (a) 2,1 (b) 2,172. In which of the following compound there are maximum no. of sp 2 hybrid Catoms:(1) Benzene (2) 1,3,5-hexatriene(2) 1,2,4-hexatriene (4) both 1 and 273. The shape of the molecule having hybrid orbitals of 20% character will be :(1) octahedral (2) tetrahedral(3) square planer (4) triangular bipyramidal74. The pH of a solution is 5. If the dilution of this solution is increased by 100 times,the pH value will be :(1) 5 (2) 7 (3) 9 (4) 875. The required amount of oxygen for combustion of 20 ml. of gaseoushydrocarbon is 50 ml. The hydrocarbon will be :(1) C 2 H 2 (2) C 2 H 4 (3) C 2 H 6 (4) C 3 H 476. The formula of Celestine is :(1) SrSO 4 (2) SrCO 3 (3) SrO (4) SrCl 277. CuCl 2 + →→Cu + Cl 2 . The required amount of electricity for this reaction is :(1) 4 faraday (2) 2 faraday (3) 1 faraday (4) 3 faraday78. Nitrogen does not forms NF 5 because :(1) The bondenergy of N≡N is very high(2) Vaccent d-orbitals are not present(3) N belongs to V group(4) There is inert effect79. The normal temperature when raised by 10 0 C, the rate of reaction will be :(1) lowered by 2 times(2) increased by 2 times(3) lowered by 10 times(4) increased by 10 times80. Which of the following gives red precipitate with ammonical cuprouschloride :(1) Propane (2) Ethane (3) Methane (4) Acetylene


www.JbigDeaL.com81. [Cu(NH 3 ) 4 ] 2+ snows the following hybridization :(1) dsp 2 (2) sp 3 d (3) dsp 3 (4) sp 382. A solution contains CI-, I - and S O 4 3- ions in it. Which of the following ion iscapable to precipitate all of above when added in this solution :(1) Pb 2+ (2) Ba 2+ (3) Hg 2+ (4) Cu 2+83. Fool’s gold is :(1) Cu 2 S (2) FeS 2 (3) Al 2 O 5 (4) CuFeS 284. In which of the following compound the central atom is in sp 2 hybrid state :(1) OF 2 (2) HgCl 2 (3) XeF 2 (4) NH 2+85. The number of alkenyl groups possible from C 4 H 7 - are :(1) 7 (2) 5 (3) 3 (4) 886. The tetraethyl lead mixed in petrol is works as :(1) Cooling agent(2) Anti knocking agent(3) Bleaching agent(4) None of these87. The alkaline hydrolysis of ester is known as :(1) dehydrogenation (2) dehydration (3) esterification (4) saponification88. The degree of ionization of 0.4 M acetic acid will be : (K a = 1.8 x 10 -5 )(1) 6.71 x 10 -3 (2) 1.6x10 -3(3) 0.4x1.8x10 -5 (4) 1.8x10 -589. Haber process is used for production of which of the following :(1) NH 3 (2) HNO 3 (3) H 2 SO 4 (4) O 390. The p ka value of phenolphthalein is 9.1 and the pH range is 8-10. In which ofthe following titrations it can be used as an indicator :(1) NH 4 OHand HCI(2) NH 4 OH and CH 3 COOH(3) NaOH and HCI(4) NH 4 OH91. Number of electrons in a one molecule of CO 2 :(1) pb 2+ (2) Hg 2+ (3) Ba 2+ (4) Cu 2+92. Which of the following species shows the maximum magnetic moment :(1) Mn +6 (2) Ni 2+ (3) Fe 3+ (4) Ag +93. K sp value of CaF 2 is 3.75 x 10 11 The solubility will be :


www.JbigDeaL.com(1) 1.45x10 -11 mol/litre -1(2)3.45x10 -4 mol/liter -1(3) 2.05x10 -4 mol/liter -1(4) 3.75 x 10 -11 mol/liter -194. When Pb 3 O 4 is heated with dilute H N O 3 it gives :(1) pbO 2 and pb(NO 3 ) 2(2) pbO and pb(NO 3 ) 2(3) pbO 2(4) pbO95. C-H bond length is least in :(1) Acetylene (2) Methane (3) Ethylene (4) Ethane96. The minimum nos. of carbon atoms in ketones which will show chainisomerism will be :(1) Seven (2) four (3) six (4) five97. Which of the following organic compound could not be dried by anhydrousCaCI 2 :(1) ethanol (2) benzene (3) chloroform (4) ethyl acetate98. Which of the following compound forms white precipitate with brominewater :(1) Nitrobenzene (2) Phenol (3) Benzene (4) all above99. Gypsum is :(1) CaSO 4 .H 2 O (2) CaSO 4 . 2H 2 O(3) 2CaSO 4 . 2H 2 O (4) CaSO 4100.Which of the following carbonium ion is most stable :+ +(1) CH 3 -C⎯CH 3 (2) CH 3 CH 2CH 3+ +(3) CH 3 0CH-CH 3 (4) CH 3


www.JbigDeaL.comANSWER SHEET1.(2) 2.(3) 3.(3) 4.(2) 5.(2) 6.(4) 7.(1) 8.(3) 9.(1) 10.(4) 11.(1)12.(1) 13.(2) 14.(4) 15.(2) 16.(4) 17.(3) 18.(2) 19.(2) 20.(2) 21.(1) 22.(2)23.(3) 24.(4) 25.(4) 26.(3) 27.(3) 28.(3) 29.(2) 30.(3) 31.(3) 32.(1) 33.(2)34.(3) 35.(2) 36.(3) 37.(2) 38.(1) 39.(4) 40.(4) 41.(3) 42(3) 43.(3) 44.(2)45.(3) 46.(3) 47.(1) 48.(1) 49.(1) 50.(1) 51.(2) 52.(1) 53.(2) 54.(3) 55.(3)56.(3) 57.(4) 58.(3) 59.(3) 60.(1) 61.(1) 62.(3) 63.(3) 64.(3) 65.(4) 66.(1)67.(3) 68.(1) 69.(1) 70.(2) 71.(3) 72.(4) 73.(4) 74.(2) 75.(1) 76.(2) 77.(2)78.(2) 79.(2) 80.(4) 81.(1) 82.(1) 83.(2) 84.(4) 85.(4) 86.(2) 87.(4) 88.(1)89.(1) 90.(3) 91.(1) 92.(3) 93.(3) 94.(1) 95.(1) 96.(4) 97.(1) 98.(2) 99.(2)100.(1)


CHEMISTRY – 1998RAJASTHAN P.E.T.1. The final product of the reaction HC = CH + 2HCI →→will be :(1) CH 2 CI-CH 2 CI (2) CH 2 =CHCI(3) CH 3 CHCI 2 (4) CHCI=CHC2. Which of the following is amphoteric :(1) GeO 2 (2) CO 2(3) PbO 2 (4) All same3. CH 3 COOC 5 H 11 is obtained by :(1) C 5 H 11 OH + CH 3 COOH(2) C 5 H 11 CH 2 OH = HCOOH(3) C 2 H 5 OH = C 5 H 11 OH(4) (CH 3 ) 3 –C-COOH = C 5 H 11 OH4. 5 amp. current is passes through a dry cell for 2 hours. The value of produced electriccurrent will be :(1) 36 x 10 8 C (2) 3.6 x 10 8 C(3) 36 x 10 4 C (4) 3.6 x 10 4 C5. Which of the following statement is false for tranis-1, 2-dichloro ethane :(1) chlorine atoms are nearer to each other(2) total nos of bonds are six(3) free rotation of C=C is possible(4) none of these6. Orthouitropnenol is a A;(1) Lewis base (2) Lewis acid(3) 1 and 2 (4) nither 1 nor 27. Which of the following shows cistrans isomerism :(1) CH 3 -C-Br=C-C1 2(2) CH 3 -CH=Ch 2(3) C1-CH=CH-CH 3(4) (CH 3 ) 2 -C=CH-C1


8. Glycine works in a reaction as :(1) Acid (2) Base (3) both 1 and 2 (4) none of these9. The true statement for 2-chlrobutane and 3- chlrobutane is :(1) First is more reactive than second(2) Second is more reactive than first(3) Chlorine atom in both are of different type(4) One name is wrong, both are same10. The magnetic moment of an ion having 4 unpaired electrons is :(1) 3.9 B.M. (2) 2.8 B.M. (3) 1.7 B.M. (4) 4.9 B.M.11. O-F bond in OF 2 compound is formed by the overlapping of following orbitals :(1) sp 2 -2p (2) sp 3 -2p (3) sp 3 -2s (4) sp-2p12. The structure of [Cu(NH 3 ) 4 ] 2+ is :(1) square planner (2) angular (3) linear (4) tetrahedral13. The no. of structural isomers of heptane is :(1) equal to pentane (2) less than hexane(3) more than pentane (4) less than pentane14. Which of the following hydroxide is soluble in NH 4 OH :(1) Sb(OH) 3 (2) Bi(OH) 3 (3) Fe(OH) 3 (4) none of above15. Which of the following differs from others :(1) Pd (2) CO (3) Ni (4) Rb16. The structure of phorone is :(1) (CH 3 ) 2 C(OH)C1 3(2) (CH 3 ) 2 C=CHCOCH=C(CH 3 ) 2(3) (CH 3 ) 2 C=CHCOCH 3(4) none of above17. Which of the following is strongest electrolyte :(1) C 12 H 12 O 11(2) H 2 O(3) CH 3 COOH(4) HI18. Which of the following statement is true :(1) O 2 2- is diamagnetic(2) O 2 + is paramagnetic(3) No is diamagnetic(4) He 2 + is less stable than He 219. For which of the following elements the quantum nos are 3, 2, 0, + ½ :(1) K (2) CO (3) Ne (4) C120. The coordination nos. of Na + and C1 - in NaCI are respectively :


(1) 6, 6 (2) 4, 6 (3) 6, 8 (4) 8, 821. In comparision of Cu and Ag :(1) Cu is easily oxidized in comparision with Ag.(2) Ag is easily oxidized in comparision with Cu(3) Both oxidizes simultaneously(4) Do not oxidizes22. Molarity of 200 ml. of 18.25 N NaOH will be :(1) 32.5 M (2) 91.25 M (3) 2.28 M (4) 22.8 M23. In Haber’s process if temperature is increased :(1) Reaction stops(2) There is no effect(3) Yield of NH 3 decreases(4) Yield of NH 3 increases24. Empirical formula of alkane, alkene and alkyne is :(1) equal to cyclopean(2) equal to each other(3) all are different(4) none of these25. CF2C1 2 is used as :(1) Anaesthic (2) Polymer (3) Refrigerant (4) Antipyretic26. The weight of carbon atom is :(1) 1.9 x 10 -23 (2) 12 gm (3) 6 gm (4) 6.02 gm. X 10 23 gm.27. The pH of 10 -8 M HCI is :(1) less than 7 (2) less than 6 (3) 8 (4) 728. Which of the following statement is true :(1) C 6 H 6 does not show resonance(2) CO 2 does not show resonance(3) Both do not show resonance(4) CO 2 and C 6 H 6 show resonating structures29. In which of the following compound >C=0 group is not present :(1) Alkane (2) Aldehyde (3) Acids (4) Ketone30. The mole fraction of acetone in a solution of 2.8 mole acetone and 8.2 mole of CHC1 3 will be :(1) 0.540 (2) 0.241 (3) 0.254 (4) 0.52431. Which of the following element has high ionization potential :(1) Ne (2) Be (3) Li (4) O32. Which of the following has highest boiling point :(1) HI (2) HC1 (3) HF (4) HBr


33. The dry ice is :(1) Solid H 2 O (2) Solid CO 2 (3) Solid & Dry H 2 O (4) none of above→34. For the reaction 2A ← C + D the value of equilibrium constant is 1 x 10 -3 . If [C] = 1.2 x 10-3 M,[D] = 3.8 x 10 -6 M the value of [A] will be :(1) 5.2 x 10 -6 M (2) 3.6 x 10 -9 M(3) 2.1 x 10 -3 M (4) 4.8 x 10 -12 M35. Which of the following does not obey the octet rule :(1) PCI 3 (2) SF 6 (3) SO 2 (4) OF 236. Mustard gas is found from :(1) C 2 H 4 & H 2 SO 4 (2) C 2 H 4 & H 2 S(3) C 2 H 4 & S 2 C1 2 (4) C 2 H 4 & CH 3 SH37. The most reactive metal is :(1) Li (2) Au (3) F (4) Pt38. Which of the following has highest melting point :(1) C 4 H 10 (2) C 3 H 8 (3) C 2 H 6 (4) CH 439. Which of the following is not a metal :(1) Au (2) Hg (3) Ag (4) none of these40. In which of the following there is strong bond :(1) C=C (2) C-C (3) C≡C (4) all same41. The shape and size of 2p, 3p, 4p and 5p orbital are :(1) only equal in d block(2) equal in s block and different in p block(3) different(4) equal42. Malachite is a ore of :(1) Cu (2) Au (3) Ag (4) Mg43. If the ionization constant of CH 3 COOH is 1.8 x 10 5 , the degree of ionization of 0.01 MCH 3 COOh will be :(1) 1.8 x 10 -7 (2) 1.8 (3) 4.2 x 10 -2 (4) 42.4 x 10 -544. If the price of Nac1 sugar are 2 and 14 rupees per kg. then the price of 1 mole NaC1 and 1mole sugar will be :(1) 7 Rs. (2) different (3) equal (4) 28 Rs.45. In which of the following there are minimum nos. of molecule :(1) 2 gm. H 2 (2) 8 gm. O 2 (3) 16 gm. CO 2 (4) 4 gm. N 246. In which of the following central atom uses sp 2 hybrid orbitals :(1) SbH 3 (2) NH 3 (3) PH 3 (4) + CH 3


63. The molecule which has linear structure is :(1) NO 2 (2) SO 2 (3) CO 2 (4) OCl 264. Which of the following have not tetrahedral geometry :(1) NH 4+(2) BF - 4 (3) SiF 4 (4) SF 41 265. N=C-C-CH2 in this compound bond⏐HBetween C(1) and C(2) is formed by hybrid orbitals of :(1) sp & sp 2 (2) sp & sp 3 (3) sp & sp (4) sp 2 -sp 266. The dipole moment of CCl 4 is zero, because of :(1) equal electron affinity of C, and Cl(2) equal size of C and Cl(3) regular size of C and Cl(4) planar structure67. The number of moles of H 2 at 500 cm.3 volume, 700 mm. pressure and 300 0 K temperaturewill be:(1) 0.203x10 -2 moles(2) 20.x10 -3 moles(3) 20.3x10 -2 moles(4) 2.03 x 10 -7 moles68.Which of the following has electronic configuration as 4f 1-14 5s 2 5p 6 5d 1 6s 2 :(1) Representative elements(2) Transition elements(3) Lanthanides(4) Actinides69. The wave number of hydrogen atom in Lymen series is 82, 200 cm. -1 . The electrongoes from :(1) III orbit to II (2) II orbit to I (3) IV orbit to III (4) none of these70. Teflen is a polymer of :(1) PVC (2) Tetrafluro ethane (3) Tetra fluro ethane (4) C 2 H 471. In which of the following s character is maximum :(1) C 6 H 6 (2) H 2 H 6 (3) C 2 H 4 (4) C 2 H 272. Benzene hexachloride is found by :(1) Addition (2) Elimination (3) Substitution reaction (4) All these73. Alkane is found by :(1) Reaction by alky l halide(2) Wurtz reaction(3) Grignard reagent(4) All these


74. The first inert gas compound invented was :(1) KrF 6 (2) XeF 6 (3) XeF 2 (4) XePtF 675. There are unpaired electrons in nitrogen according to :(1) Hund’s rule(2) Aufabu’s principal(3) Paulis principal(4) none of these76. Which of the following is smallest in size :(1) Na + (2) F - (3) N - 3 (4) O -277. The wave character of electron was invented by :(1) Schrödinger (2) Henisber (3) Niel Bohr (4) Davisson & Germer78. The electronic configuration of Chromium will be :(1) [Ar] 3d 5 4s 3 (2) [Ar]3d 4 4s 2 (3) [Ar] 3d 5 4 s 1 (4) [Ar] 3d 5 4s 079. In which of the following nos. of primary carbon atoms are maximum :(1) is pentane (2) iso-octane (3) neopentane (4) all of these80. Na 2 S 2 O 3 is used in photography because :(1) It is a compound of sulphur(2) It reacts with Ag Br to form sodium silver thisulphate(3) It is an antichlor reagent(4) none of these81. Borax is found in :(1) Punjab (2) Rajasthan (3) Utterpradesh (4) Delhi82. Which of the following is not true for O 3 :(1) it converts into colourless liquid when condensed(2) it converts into violet black solid when it condensed(3) it is blue gas(4) it is a allotrople of oxygen83. H 2 O and D 2 O both have :(1) common chemical properties(2) different physical and chemical properties(3) common physical but different chemical properties(4) common physical properties84. Which of the following is not a conjugate base :(1) CH - 3 (2) OH - (3) CO 2 - (4) none of these85. Plaster of paris is a compound of the following element :(1) K (2) Ca (3) Mg (4) Na86. Benzene → Toluene is formed by :(1) Anti-mark rule


(2) F.C.R.(3) Wurtz reaction(4) Markownikoff’s rule87. The frequency of wave of 4000 Ǻ wave. Length will be :(1) 7.5 x 0 2 s -1 (2) 75 x 10 10 s -1 (3) 7.5 x 10 14 (4) 0.75 x 10 2 s -188. The oxidation no. of C in CO 2 is :(1) +1 (2) +2 (3) +4 (4) 089. H 2 O 2 is :(1) strong oxidizing agent and weak reducing agent(2) neigther oxidizing agent nor reducing agent(3) only reducing agent(4) only oxidizing agent90. Which element have maximum oxidation states :(1) Sc (2) Zn (3) B (4) Mn91. Carborundum is :(1) SiB (2) SiC (3) SiO 2 (4) CO 292. Stainless steel is :(1) Fe, Ni, CO, C (2) Fe, Mg, Ni, C(3) Fe, Cr, Ni, C (4) Fe, Mn, Cr, Ni93. fluorine is formed by electrolysis of the fused mixture of K and HF because :(1) It is most reactive (2) It is a gas(3) It is strong oxidizing agent (4) It is (F 2 ) toxic94. Which of the following Lewis acid is strongest :(1) BI 3 (2) BCI 3 (3) BF 3 (4) BBr 395. The colour of the solution of alkali metal in liquid ammonia appears to blue due to :(1) Ammonical metal ion and electron(2) Ammonical electron(3) Ammonical metal ion(4) Metal ion96. The solubility product of calcium oxalate is 2.5 x 10-3 mole2/liter-2 . The requiredminimum concentration of calcium ion to precipitate it will be :(1) >5x10 -2 (2) 5x10 -2 (3)


(4) Bauxite99. Which of the following gives rod colour precipitate with sodium cupritartaarate :(1) CH 3 COOH (2) CH 3 COCH (3) CH 3 COC 2 H 5 (4) CH 3 CHO100. Which of the following are present in picric acid :(1) –NO 2 group(2) –OH and –NO 2 group(3) –NO 2 and –COOH groups(4) –OH groupANSWER SHEET1.(3) 2.(1) 3.(1) 4.(4) 5.(3) 6.(1) 7.(3) 8.(3) 9.(4) 10.(4) 11.(2)12.(4) 13.(3) 14.(3) 15.(4) 16.(2) 17.(4) 18.(2) 19.(1) 20.(1) 21.(1) 22.(2)23.(3) 24.(3) 25.(3) 26.(1) 27.(1) 28.(2) 29.(1) 30.(3) 31.(1) 32.(3) 33.(2)34.(3) 35.(2) 36.(3) 37.(1) 38.(1) 39.(4) 40.(3) 41.(3) 42.(1) 43.(3) 44.(2)45.(4) 46.(4) 47.(3) 48.(3) 49.(1) 50.(3) 51.(4) 52.(1) 53.(3) 54.(1) 55.(3)56.(1) 57.(3) 58.(2) 59.(1) 60.(2) 61.(3) 62.(2) 63.(3) 64.(4) 65.(1) 66.(3)67.(4) 68.(3) 69.(2) 70.(3) 71.(4) 72.(1) 73.(4) 74.(4) 75.(1) 76.(1) 77.(1)78.(3) 79.(2) 80.(2) 81.(3) 82.(4) 83.(3) 84.(4) 85.(2) 86.(2) 87.(3) 88.(2)89.(1) 90.(4) 91.(2) 92.(3) 93.(3) 94.(1) 95.(2) 96.(1) 97.(2) 98.(1) 99.(4)100.(2)


Mathematics


RAJASTHAN P.E.T.MATHS-19981. All letters of the word ‘CEASE’ are arranged randomly in a row then the probability that two Eare found together is :(1) 7 (2) 3 (3) 2 (4) 15 5 5 52. Three numbers are selected randomly between 1 to 20. Then the probality that they areconsecutive numbers will be :(1) 7 (2) 3 (3) 5 (4) 1190 190 190 33. If the four positive integers are selected randomly from the set of positive stegers then theprobability that the number 1, 3 , 7, 9 are in the unit place in the product of 4 digitsosetected is :(1) 7 (2) 2 (3) 5 (4) 16625 5 625 6254. If the position vectors of the vertices A, B, C are 6i, 6j, k respectively w.r.t. origin O then thevolume of the tetranedron OABC is :(1) 6 (2) 3 (3) 1 (4) 16 3∧∧∧∧∧∧∧∧∧∧∧∧5. If three vectors 2i – j - k, i + 2j – 3k, 3i + λλ λj + 5 k are coplanar then the value of λλis :(1) – 4 (2) – 2 (3) – 1 (4) 0∧∧∧∧∧∧6. The vector perpendicular to the vectors 4i, - j + 3k and – 2i + j - 2k whose magnitude is 9 :∧ ∧ ∧ ∧ ∧ ∧ ∧ ∧ ∧(1) 31 + 6j – 6k (2) 31 – 6j + 6k (3) – 3i + 6j + 6k (4) none of these7. The area of the region bounded by the curves x 2 + y 2 = 8 and y 2 = 2x is :(1) 2π + 1 (2) π + 1 (3) 2π + 4 (4) π + 43 3 3 3ππ8. The value of 0 log (1 + cos x) dx is :(1) - π log 2 (2) π log 1 (3) π log 2 (4) π log 22 2 249. The value of 3 √√(4 – x) (x – 3) dx is :(1) π (2) π (3) π (4) π16 8 4 2∧∧∧∧∧∧∧∧∧∧∧∧∧∧∧∧


10. The value of dx is :x(x n + 1)(1) 1 log x n + cn x n + 1(2) log x n + 1 + cx n(3) 1 log x n + 1nx n(4) log x n + cx n + 111. The value of cos (log x) dx is :(1) 1 [sin(log x) + cos (log x)] + c2(2) x [sin(log x)] + cos(log x)] + c2(3) x [sin(log x) – cos(log x)] + c2(4) 1 [sin(log x) – cos(log x)] + c212. The value of e x (1 + sin x ) dx is :( 1 + cos x)(1) 1 e x sec x + c (2) e x sec x + c2 2 2(3) 1 e x tan x + c (4) e x tan x + c2 2 213. The value of 1 is dx :3 sin x – cos x + 3(1) tan -1 tan x + 1 + c2


(2) 1 tan -1 2 tan x + 1 + c2 2(3) tan -1 2 tan x + 1 + c2(4) 2tan -1 2 tam x + 1 + c214. Divide 10 into two parts such that the sum of double of the first and the square of the secondis minimum :(1) 6,4 (2) 7,3 (3) 8, 2 (4) 9,115.. The value of sin 2x dx is ;sin 4 x + cos 4 x(1) tan -1 (cot 2 x) + c (2) tan -1 (cos 2 x) + c(3) tan -1 (sin 2 x) + c (4) tan -1 (tan 2 x) + c16. The value of √ 1 + sec x dx is :(1) 1 sin-1 (√2 sin x) +c(2) – 2sin-1 (√2 sin x/2) + c(3) 2sin-1 (√2 sin x ) + c(4) 2sin-1 (√2x/2) + c17. The value of (x 2 + 1 ) dx is :x 4 + x 2 + 1(1) 1 tan -1 x – 1/x + c√3√3(2) 1 log (x – 1/x) - √3 + c2√3 ( x – 1/x) + √3


(3) tan -1 x + 1/x + c√3(4) tan -1 x – 1/x + c√3118. The value of x 2 ( 1 – x 2 ) 3/2 dx is :0(1) 1 (2) π (3) π (4) π32 8 16 32∞19. The value of xdx is :0 ( 1 + x ) ( x 2 + 1 )(1) 2π (2) π (3) π (4) π16 3220. y 2 = 8x and y = x(1) 64 (2) 32 (3) 16 (4) 83 3 3 321. If in a triangle ABC , O and O′ are the incentre and orthocenter respectively then (OA + OB+ OC) is equal to :→ → → →(1) 20′0 (2) O′0 (3) OO′ (4) 200′→ → → → → → → →22. If a + b + O = a and a = 5 b = 3, c = 7 then angle between a and b is :(1) π (2) π (3) π (4) π2 3 4 623. i.(j k) + j.(k x i) + k.(j x i) is equal to :(1) 3 (2) 2 (3) 1 (4) 024. One card is drawn at random from a pack of playing cards the probability that it is an ace orblack king or the queen of the heart will be :(1) 3 (2) 7 (3) 6 (4) 152 52 52 5225. 15 coins are tossed then the probability of getting 10 heads tails will be :(1) 511 (2) 1001 (3) 3003 (4) 300532768 32768 32768 32768


26. The odds against solving a problem by A and B are 3 : 2 and 2 : 1 respectively then theprobability that the problem will be solved is :(1) 3 (2) 2 (3) 2 (4) 115 15 5 1527. The pole of the line ιx = my +n =0 w.r.t. the parabola y 2 =4ax will be :(1) -n , - 2am (2) -n , 2 am1 1 1 1(3) n , -2am (4) n , 2am1 1 1 128. If 2x + y + λ = 0 is normal to the parabola y 2 = 8x then λ is :(1) -24 (2) ≠ 8 (3) -16 (4) 2429. If the line ιx = my + n = 0 is tangent to the parabola y 2 = 4ax then :(1) mn= aι 2 (2) ιm=an 2 (3) ιn=am 2 (4) none of therse30. f: R→ R, f(x) = x ⏐x ⏐ will be :(1) many one onto (2) one one onto(3) many are into (4) one one into31. lim (sec x – tan x) is equal to :x→π/2(1) 2 (2) -1 (3) 1 (4) 0log(1+2ax)-log(1-bx) ,32. If f(x) x x≠0K , x=0Is continuous at x = 0 then value of K is :(1) b + a (2) b – 2a (3) 2a – b (4) 2a + b33. If f(x) = x - 3 then f′ (3) is :(1) -1 (2) 1 (3) 0 (4) does not exist34. If tan x = 2t and sin y = 2t then the value of dy is :1 – t 2 1 + t 2 dx(1) 1 (2) t (3) 1 ___ (4) 1___1 – t 1 + t35. If x p + y q = (x + y) p+q then dy is :


dx(1) – x (2) x (3) – y (4) yy y x x36. All the points on the curve y 2 = 4a[ x + a sin (x)], where the stangent is parallel to the axis ofx are lies on :a(1) circle (2) parabola (3) stright line (4) none of these37. The length of normal at any point to the curve y = c cos h (x/c) is :(1) fixed (2) y 2 (3) y 2 (4) yc 2 c c 238. The weight of right circular cylinder of maximum volume inscribed in a sphere of diameter2a is:(1) 2√3a (2) √3a (3) 2a (4) a__√3 √339. The intercept of the latus rectum to the parabola y 2 = 4ax b and k then k is equal to :(1) ab (2) a (3) b ___ (4) ____ab___a - b b - a b - a b - a40. The equation of directris to the parabola 4x 2 – 4x – 2y + 3 = 0 will be :(1) 8y= 9 (2) 8x= 9 (3) 8y=7 (4) 8x= 741. If f(x) = 2 x + 2 x then f(x + y). f(x-y) is :2(1) 1[f(2x) – f(2y)] (2) 1[f(2x) – f(2y)]4 2(3) 1[f(2x) + f(2y)] (4) 1[f(2x) + f(2y)]4 242. The period of cos x will be :(1) π (2) π (3) π (4) 2π4 243. lim 3 x - 1 is equal to :x→x(1) 2 log 3 (2) 3 log 3 (3) log 3 (4) none of these44. If f(x) = x sin (1 / x) , x ≠ 00 , x=0at then at x = 0 the function f(x) is :(1) differentiable (2) differentiable (3) continuous but not differentiable (4) none of these


45. Differential coefficient of e sin – 1 x w.r.t. sin -1 x is:(1) sin -1 x (2) esin -1 x (3) ecos -1 (4) cos -1 x46. If y = tan -1 3a 2 x - x 3 then dy is :a (a 2 – 3x 2 ) dx(1) 3a 2 __ (2) 3a____a 2 + x 2 a 2 + x 2(3) a (4) 3____a 2 + x 2 a 2 + x 247. The angle of intersection between xy = a 2 , x 2 + y 2 = 2a 2 is :(1) 90 0 (2) 45 0 (3) 30 0 (4) 0 048. The length of the subtangent to the curve x m y n = a m+ n is propoteional to :(1) x 2 (2) y 2 (3) y (4) xyx49. The st. line x + y = 2 is tangent to the curve (x ) n + (y ) n = 2 at the point (a,b) then n is :a b a b(1) any real number (2) 3 (3) 2 (4) 150. If α, β are the roots of the equation x 2 – 2x cos θ + 1 = 0 then equation whose roots are α n/2 ,β n/2 will be :(1) x 2 – 2x cos (nθ) + 1 = 0(2) x 2 – 2nx cos(nθ) +1 = 0(3) x 2 – 2x cos(2nθ) +1 = 0(4) x 2 – 2x cos nθ + 1 = 0251. 33th exponents of the eleventh roots of unity will be :(1) 1 (2) -11 (3) 0 (4) 1152. If sin α + sin β + sin γ = 0 cos α + cos β + cos γ then sin 2 α + sin 2 β + sin 2 γ is equal to :(1) 2_ (2) - 3_ (3) 3 (4) 03 2 253. sec h-1 (1/2) is :(1) log (√3 ± √2) (2) log (√3 ± 1) (3) log (2 ± √3) (4) none of these54. The imaginary part of (x + iy) is :(1) 1 cos h 2x cos 2y (2) 1 cos 2x cosh h 2y2 2(3) 1 sin h 2x sin 2y (4) 1 sin 2x sin h 2y


2 255. The image of the point (- 1, 2) in the st. line x – 2y = 3 is :(1) 9 , - 23 (2) 11 , -22 (3) 13 , -21 (4) (3, -4)5 5 5 5 5 556. The locus of the middle point of the intercept made by x cos α + y sin α = p on axes is :(1) x 2 + y 2 = p 2 (2) x 2 +y 2 =4p -2 (3) x 2 +y 2 = p 2 (4) x 2 +y 2 =4p -257. The locus of the middle point of the chord of length 2ι to the curve x 2 + y 2 = a 2 will be:(1) x 2 +y 2 =a 2 ι 2(2) 2x 2 +2y 2 =ι+a 2(3) x 2 +y 2 = ι 2 +a 2(4) 2x 2 +2y 2 = a 2 -ι 258. The equation of the circle whose diameter is common chord to the circles x 2 +y 2 +2ax +c= 0and x 2 +y 2 +2by+c= 0 is:(1) x 2 +y 2 - 2ab 2 x + 2a 2 by + c = 0a 2 +b 2 a 2 +b 2(2) x 2 +y 2 - 2ab 2 x - 2a 2 by + c = 0a 2 + b 2 a 2 +b 2(3) x 2 +y 2 + 2ab 2 x + 2a 2 by + c = 0a 2 +b 2 a 2 +b 2(4) x 2 +y 2 + 2ab 2 x - 2a 2 by + c = 0a 2 +b 2 a 2 +b 259. If (3, λ) and 5,6) are the conjugate points to the curve x 2 +y 2 = 3 then λ is :(1) -1 (2) 1 (3) -2 (4) 260. The equation of the pair of tangents at (0,1) to the circle x 2 + y 2 – 2x -6y +6 = 0 is:(1) 3(x 2 -y 2 )+4xy-4x-6y+3=0(2) 3y 2 +4xy-4x-6y+3=0(3) 3x 2 +4xy-4x-6y+3=0(4) 3(x 2 +y 2 )+4xy-4x-6y+3=061. The amplitude of 1+cos θ + i sin θ 2 is :1+cos θ - i sin θ(1) - nθ (2) - nθ (3) nθ (4) nθ2 23/ 862. The product of all roots of 1 + i √3 is:2 2


(1) 2 (2) -1 (3) 0 (4) 163. If coshα = sec x then tan 2 x/2 is :(1) cos2 (α/2) (2) sin2 α/2 (3) cot2 (α/2) (4) tan h2 α/264. The real part of the principle value of 2 -i is :(1) sin (log 2) (2) cos (1/log2) (3) cos [log (1/2)] (4) cos (log2)65. The two vertices of triangle are (2, - 1), (3, 2) and the third vertex lies on x + y = 5. The areaof the triangle is 4 units then the third vertex is :(1) (0,5) or (1,4) (2) (5, 0) or (4, 1) (3) (5, 0) or (1, 4) (4) (0, 5) or (4, 1)66. If 2 a+ b + 3c = 0 than the line ax + by + c = 0 passes through the fixed point that is:(1) 2 , 1 (2) 0, 1 (3) 2 , 0 (4) none of these3 3 3 367. Straight lines ax ± by ± c = 0 represent a :(1) Rhombus (2) Square (3) Rectangle (4) None of these68. The equation of the circle passing through (2a, 0) and whose radical axis w.r.t. the circle x 2 +y 2 = a 2 is x = a will be :2(1) x 2 +y 2 +2ay=0(2) x 2 +y 2 +2ax=0(3) x 2 +y 2 -2ay=0(4) x 2 +y 2 -2ax=069. The circles x 2 +y 2 +2ax+c=0 and x 2 +y 2 +2by+c=0 touches each other then:(1) a 2 +b 2 =c2 (2) 1 + 1 = 1 (3) 1 + a = 1 (4) 1 - 1 = 1a 2 b 2 c 2 a 2 b 2 c a 2 b 2 c70. The pole of the polar w.r.t. the circle x 2 +y 2 = c 2 lies on x 2 +y 2 = 9c 2 then this polar is tangent toconcentric circle whose equation will be :(1) x 2 +y 2 = 4c 2 (2) x 2 +y 2 = c 2 (3) x 2 +y 2 = 9c 2 (4) none of these9 471. In a G.P. (m + n) th the term is a and (m-n) th term is 4 then mth term will be :(1) -6 (2) 1/6 (3) 6 (4) none of these72. The sum of n terms of 1 + 3 + 7 + 15 + … is :


2 4 8 16(1) 2n-2+2 n (2) 1-n + 2 n (3) n 2 -n (4) n – 1 + 2-n73. If 10 points lie on a plane out of which 5 are on a st-line, then total number of trianglesformed by them are :(1) 120 (2) 110 (3) 150 (4) 10074. If (1+x) n = C 0 + C 1 x + C 2 x 2 + ….+ C n x n then value of C 0 + C 1 + C 2 +….+2 3 4C n is :n + 2(1) 2 n + 1 (2) n2 n+1 _____(n+1) (n+2)(n+1) (n+2)(3) n2 n+1 (4) n2 n+1 _____(n+1) (n+2)(n+1) (n+2)75. The square roots of 1 + 2x + 3x 2 + 4x 3 + … is :(1) (1-x) -1 (2) (1+x) (3) 1+x) (1-x)76. If (1+x) n = C 0 + C 1 x + C 2 x 2 +…..then C 0 + C 1 + C 2 + ….:2 3(1) 2 n+1 + 1 (2) 2 n-1n + 1 n - 1(3) 2 n+1 + 1 (4) 2 n+1n + 1 n + 177. 2 ac - b 2 a 2 c 2ac 2 2 ab - c 2 b 2c 2 b 2 2 b c - a 2(1) (a 3 + b 3 + c 3 – 3abc) 2(2) (a 2 + b 2 + c 2 ) 3(3) (ab + bc + ca) 3(4) (a + b + c) 678. If for any two square materscies A and B, AB= A, BA= B than A 2 :(1) B 2 (2) adj A (3) B (4) A79. 1 3 6If A 3 5 1 then adj. A is :5 1 3


(1) 14 4 - 224 -22 1422 -14 4(2) 14 4 - 224 -22 14-22 14 4(3) - 14 4 224 22 -1422 - 14 4(4) 14 - 4 - 22- 4 -2 2 14-22 14 - 480. The A.M. of any two numbers is 16 and their H.M. = 63 then their G.M. will be :4(1) √3 (2) 6 √3 (3) √7 (4) 6 √781. The sum of n terms of 1.2.3 + 2.3.4 will be :(1) n ( n + 1 ) ( n + 2 ) ( n + 3 )4(2) 2n ( n + 1 ) ( n + 2 ) ( n + 3 )3(3) ( n + ) ( n + 2 ) ( n + 3 )4(4) n ( n – 1 ) ( n – 2 ) ( n – 3 )482.Out of 14 players there are 5 bowlers. Then the total number of ways of selecting a team of 11players of which at least 4 are bowlers are :(1) 275 (2) 264 (3) 263 (4) 26583. If ( 1 + x) n = C 0 + c 1 x + C 2 x 2 + ….+ C n x n then the value of C 1 + 2C 2 + 3C 3 + 4C 4 + ….+ nC nwill be :(1) 2 n-1 (2) n . 2 n-1 (3) 2 n (4) 084. If the coefficients of the second third and fourth terms in the expansion of ( 1 + x) 2n are inA.P. then 2n 2 – 9n is :


(1) - 14 (2) 14 (3) -7 (4) 785. If a - b -c-a b -c + λ abc = 0 then λ is :-a -b c(1) -2 (2) 2 (3) 4 (4) -486. If A = 2 3 and B = 1 21 2 3 3 then :2 4(1) BA= 4 7 (2) BA= 4 9 89 15 7 15 148 14(3) AB= 8 15 12 (4) AB= 8 44 9 10 15 912 1087. If A = 1 k then A n =0 1(1) n nk (2) n k n0 n 0 n(3) 1 nk (4) 1 k n0 1 0 188. ( 1 – i ) ( 1 + 2i ) ( 2 – 3i ) =(1) √130 (2) √13 (3) 130 (4) 1389. (a + b ) (aω + bω 2 ) (aω 2 + bω) =(1) 6 (a 2 +b 3 ) (2) 3 (a 3 +b 3 ) (3) a 3 +b 3 (4) 0


90 If z - 2 > z – 4 then the correct statement is :(1) x > 3 (2) x > -3 (3) x . 1 (4) x > -191. If α, β are the roots of the equation x2 – 5x – 3 = 0 then the equation whose roots are1 , 1 will be :2 θ - 3 2 β - 3(1) 33x 2 + 4x + 1 = 0 (2) 33x 2 – 4x – 1 = 0(3) 33x 2 +4x + 1 = 0 (4) 33x 2 + 4x – 1 = 092. If x is real then the values ofx 2 + 14x + 9 is :x2 + 2x + 3(1) ( - ∞, - 5 ) ∪ ( 4, ∞ ) (2) [ - 5, 4] (3) [-4, 5] (4) [4, 5]93. The sun of numbers divisible by 7 and lies between 100 to 300 will be :(1) 5486 (2) 8588 (3) 5086 (4) 558694. The area of the triangle represent by z, iz, and z – iz will be :(1) 2 z 2 (2) z 2 (3) z 2 (4) 02_ _95. If z = x + iy then zz + 2(x + z) + c = 0 will represent :(1) a point (2) parabola (3) st-line (4) circle96. If x = 2√3i then x 4 + 4x 2 – 8x + 39 is equal to :(1) -20 (2) -52 (3) – 20 + 16i√3 (4) 20+16i√97. If one root of the equation 2x 2 – bx + c = 0 is square of the other then :(1) b 2 – 4ac = θ (2) ac (a + c + 3b) = b 3(3) ac = b 3 (4) none of these98. (a – b) 2 , (b – c) 2 , (c – a) 2 are in A.P. the 1 , 1 , 1____will be : a – b b – c c - a(1) in H.P. (2) in G.P. (3) in A.P. (4) none of these99. If the first term of an infinite G.P. scries is 1 and its every term is the sum of the nextsuccessive terms then fourth term will be :(1) 1_ (2) 1 (3) 1 (4) 116 8 4 2100. Correct statement is :(1) (AB) -1 = B -1 A -1 (2) (AB) T = A T B T (3) (AB) -1 = A -1 B -1 (4) none of these


ANSWER SHEET1.(3) 2.(2) 3.(4) 4.(1) 5.(1) 6.(3) 7.(3) 8.(2) 9.(2) 10.(1) 11.(2)12.(2) 13.(3) 14.(4) 15.(4) 16.(4) 17.(1) 18.(4) 19.(4) 20.(2) 21.(3) 22.(2)23.(1) 24.(2) 25.(3) 26.(1) 27.(1) 28.(1) 29.(3) 30.(4) 31.(4) 32.(4) 33.(4)34.(1) 35.(4) 36.(3) 37.(3) 38.(3) 39.(4) 40.(3) 41.(2) 42.(3) 43.(3) 44.(2)45.(2) 46.(1) 47.(4) 48.(3) 49.(1) 50.(1) 51.(4) 52.(3) 53.(3) 54.(4) 55.(2)56.(4) 57.(1) 58.(3) 59.(3) 60.(2) 61.(4) 62.(4) 63.(4) 64.(4) 65.(3) 66.(1)67.(1) 68.(3) 69.(4) 70.(2) 71.(3) 72.(4) 73.(3) 74.(4) 75.(1) 76.(3) 77.(1)78.(4) 79.(4) 80.(4) 81.(3) 82.(2) 83.(2) 84.(3) 85.(3) 86.(1) 87.(3) 88.(1)89.(3) 90.(1) 91.(4) 92.(2) 93.(4) 94.(3) 95.(4) 96.(3) 97.(2) 98.(1) 99.(2)100.(1)


RAJASTHAN P.E.T.MATHS-19951. The equation of the normal to the circle x2 + y2 = a2 at point (x′ y′) will be :(1) x′y - xy′ = 0 (2) xx′ - yy′ = 0(3) x′y + xy′ = 0 (4) xx′ + yy′ = 02. Equation of the bisector of the acute angle between lines 3x + 4y + 5 = 0 and12x – 5y – 7 = 0 is :(1) 21x + 77y + 100 = 0(2) 99x – 27y + 30 = 0(3) 99x + 27y + 30 = 0(4) 21x – 77y – 100 = 03. Equation to the line passing through the point (-4,5) and perpendicular to 3x= 4y = 7 :(1) 3x-4y+32=0 (2) 4x+3y+1= 0(3) 3x+4y-8=0 (4) 4x-3y+31=04. If θθis the angle between two straight lines represented by ax 2 + 2hxy + by 2 =0 then :(1) tan θ = 2√h 2 + aba + b(2) cos θ = 2√h 2 – aba + b(3) tan θ = √h 2 – aba + b(4) tan θ = 2√h 2 – aba + b5.The real part of cos h ( αα+ iββ β) :(1) sin α sin hβ (2) cos α cos hβ(3) 2 cos nθ (4) cos hα cos β6. If z = cos θθi sin θθ θ, then the value of z n + 1 will be :z n(1) sin 2nθ (2) 2 sin nθ (3) 2 cos nθ (4) cos 2nθ7. If ααand ββare the roots of the equation x 2 – 2x + 4 = 0 then the value of ααn + ββwill be :(1) i2 n+1 sin (nπ/3) (2) 2 n+1 cos (nπ/3)(3) i2 n-1 sin (nπ/3) (4) 2 n-1 cos (nπ/3)8. [sin (ααα + θθ θ) – e ai sin θθ θ] n is equal to :(1) cos n α e inθ (2) sin n α e inθ(3) cos n α e- inθ (4) sin n α e -inθβ n


9. If A is a skew symmetric matrix of second order and C is a column matrix ofsecond order then CAC is equal to :(1) [0] (2) [1] (3) 0 1 (4) 1 01 0 0 110. If A = 3 1 and I = 1 0 then the correct statement is :-1 2 0 1then the correct statement is :(1) A 2 +5A – 7I = 0(2) –A 2 +5A+7I =0(3) A 2 -5A+7I = 0(4) A 2 +5A+7I = 011. If A and B are the two matrices of the same order and A 2 -b 2 = (A+B) (A-B) , thenthe correct statement will be :(1) A′B′ = AB (2) AB=BA (3) A 2 +B 2 = A 2 -B 2 (4) none of these12. The value of the determinant a-b-c 2 a 2 a2 b b-c-a 2 b will be :2 c 2 c c-a-b(1) (a-b-c) (a 2 +b 2 +c 2 ) (2) (a+b+c) 3 (3) (a+b+c)(ab+bc+ca) (4) none of these13. If (1 + x) n = C 0 + C 1 x + C 2 x 2 + …+ C n x n , then C 0 -C 1 +C 2 -C 3 +….+(-1) n C n is equalto:(1) 3 n (2) 2 n (3) 1 (4) 014. The term independent of x in the expansion of x + 1x is :(1) 1.3.5……..(2n-1) . 2 n-1n!(2) 1.3.5………(2n-1) . 2 nn!(3) a.3.5……(2n-1) .2 n(4) none of these15. (1 – x) 3 is equal to :(1) x 3 +3x 2 +3x-1 (2) x 3 -3x 2 +3x-1(3) x 3 -3x 2 -3x+1 (4) x 3 +3x 2 +3x+12n16. If n ∈∈N, then m2 is equal to :nm=1(1) m(m+1)(2m+1)6


(2) n(n-1)(2n-1)6(3) m((m-1)(2m-1)6(4) n(n+1)(2n+1)617. If A.M. and H.M. between two numbers are 27 and 12 respectively then theirG.M. is:(1) 9 (2) 18 (3) 24 (4) 3618. If 1 , 1 , 1 , are in A.P. then :q + r r + p p + q(1) p2,q2, r2 are in A.P.(2) p,q,r are in A.P.(3) p,q,r are in G.P.(4) 1 , 1 , 1 are in A.P.p q r19. If ααand ββare the roots of the equation x 2 – ax + b = 0 and v n = ααn + ββn then :(1) v n+1 = av n + bv n-1(2) v n+1 = bv n – av n-1(3) v n+1 = av n – bv n-1(4) v n+1 = bv n + av n-120. If ααand 1 are the roots of the equation 5x 2 +13x+k = 0 then k will be:αα(1) 5 (2) – 5 (3) 13 (4) 121. The value i 3 -i 5 -i 10 -i 16 will be :(1) 0 (2) i (3) – 2 – 2i (4) 2 – 2i22. A coin tossed m + n (m > n) , times then the probability that the head appears mtimes continuosly is :(1) m + n (2) n + 2 (3) m ___ (4) m + 22 m+n 2 m+1 2 m+n 2 n+123. For any two events A and B if P(A ∪∪B) = 5/6, P(A ∩∩B) = 1/3, P(B) = ½ thenP(A) is :(1) ½ (2) 2/3 (3) 1/3 (4) none of these24. If M and N are any two events , then the probability of happening exactly oneevent is:(1) P(M) + P(N) – P(MN)(2) P(M) + P(N) – 2P(MN)⎯ ⎯ ⎯(3) P(M) + P(N) + 2P(MN)


(4) none of these25. A bag contains 3 white and 5 black balls. One ball is drawn at random. Then theprobability that it is black is :(1) 1 (2) 3 (3) 5 (4) 38 8 8 526. A box contains 100 bulbs, out of these 10 are used. 5 bulbs are choosen atrandom. Then the probability that no one is fused is :(1) 9105(2) 90 C 5 (3) 1100 C 5 25(4) 10 -527. For any two events A and B the correct statement is :(1) P (A ∩ B) ≤ P (A) + P (B)(2) P (A ∩ B) ≤ P (A) + P (B) -1(3) P (A ∩ B) ≥ P (A) + P (B) -1(4) P (A ∩ B) ≥ P (A) + P (B)→→28. For any non zero vector a the correct statement is :→ → → → → → → →(1) a . a ≤ 0 (2) a . a = 0 (3) a . a > 0 (4) a. a ≥ 0→→→→→→29. a . (b x c ) = 0 then the correct statement is :→ → →(1) out of a, b , c any two vectors are parallel→ → →(2) a, b, c are coplanar→ → →(3) any two are equal a , b, c(4) at least one above statement is correct→→→→→→→→30. If A x B = 0 where A and B are non zero vectors then :→ →(1) A and B are perpendicular to each other→ →(2) the angle between A and B is π→ →(3) A and B parallel vectors→(4) B is unit vector∧∧∧∧∧∧∧∧∧∧∧∧


31. If 2i + j - k and i – 4j + λλ λk are perpendicular to each other then λλis equalto:(1) – 3 (2) – 2 (3) – 1 (4) 032. If d__ φφ(x) = f(x) then f(x) dx is equal to :1dx2(1) f(1) – f(2) (2) φ(1) - φ(2) (3) f(2) –f(1) (4) φ(2) - φ(1)a33. If f (a – x) = f(x), then xf(x) dx is equal to :0a a/2 a(1) f(x)dx (2) a f(x)dx (3) a f(x)dx (4) none of these0 0 0a34. f(x)dx = 2 f(x)dx when :-a 0a(1) f(2a-x) = - fx (2) f(2a-x)=f(x) (3)f(-x)=-f(x) (4) f(-x)=f(x)235. | 1 – x|dx is equal to :0(1) 0 (2) 1 (3) 3 (4) 12 2ππ36. For any integer n the value of e cos2 cos 3 (2n+1)x dx will be:0(1) e 2 (2) 0 (3) 1 (4) e37. sin 2x dx is equal to :sin 4 x + cos 4 x(1) 2 tan -1 (tan 2 x ) + C(2) tan -1 (x tan 2 x) + C(3) tan -1 (tan 2 x) + C(4) none of these38. 1 dx is equal to :x 5 (1) - 1 + C (2) - 1 + C (3) – 1 + C (4) -5 + C5x 4 5x 6 4x 4 x 6


39. The function sin x + cos x is maximum when x is equal to :(1) π (2) π (3) π (4) π6 4 3 240. If the normal to a curve is parallel to axis of x, then the correct statementis :(1) dx = - 1 (2) dx (3) dx = 0 (4) dy = 0dy dy dy dx41. d sin -1 x is equal to :dx(1) - 1 (2) 1 (3) 1 __ (4) - 1____√x 2 – 1 √x 2 – 1 √1-x 2 √1-x 242. The differential coefficient of e x-3 is :(1) 2x 3 e x3 (2) 3x(e x3 ) (3) e x3 (4) 3x 2 e x343. d (x x ) is equal to :dx(1) x x log (e/x) (2) x x log ex (3) log ex (4) x x log x44. lim [f(x),g(x)] will exist, when :x→→→ a(1) lim f(x) is existsx→ag(x)(2) lim [f(x)] g(x) is existsx→a(3) lim f (x) or lim g(x) is existsx→a(4) lim f (x) and lim g(x) both existsx→ax→a45. lim sin x is equal to :x→→ →0x(1) 2 (2) – 1 (3) 1 (4) 046. If f(x) = sin [x] , [x] ≠≠0 where [x] is a greatest integer less or equal to x thenlim f(x) is equal to :x→→ →0(1) – 1 (2) 0 (3) 1 (4) does not exist


47. If A = {-2, -1, 0, 1,2} and f:A→→→ R such that f(x) = x 2 + 1, then the range of f willbe:(1) {1, ± 2, ±5} (2) {1,2,5} (3) {-2, -1, 0, 1,2} (4) none of these48. The point (at3, at2) will lies on the curve :(1) x 3 = ay 2 (2) x 2 = ay (3) y 2 = ac (4) y 3 = ax 249. The diameter of the circle x 2 + y 2 + 4x – 6y = 0, is :(1) √52 (2) √13 (3) √26 (4) √2050. The pole of the line ττ τx + my + n = 0 w.r.t. the circle x 2 + y 2 = a 2 is :(1) - n a 2 , - n a 21 m(2) - a , mna 2 ma 2(3) - 1 a 2 , m a 2n n(4) - 1 a 2 , - m a 2n n51. Two dice thrown together then the probability of getting a sum of 7, is :(1) 7 (2) 6 (3) 5 (4) 8_36 36 36 36⎯⎯52. For any two events A and B, P(A∩∩ ∩B) is equal :∩(1) P(A) – P(A∩B) (2) P(A) – P(A ∩B)⎯ ⎯(3) P(A) – P(A∪B) (4) P(A) + (A ∩ B)⎯⎯⎯⎯53. If A and B are two events, then P(A / B) is equal to :⎯ ⎯(1) P(A) /P(B) (2) 1-P(A+B)⎯P(B)(3) 1- P(AB) (4) 1- P(A/B)


P(B)54. If A ≤≤B, then B ∪∪A will be :(1) [0] (2) φ (3) A (4) B⎯⎯55. P A is equal :A ∪∪B⎯ ⎯ ⎯(1) P(A) (2) P(A ∩ B) (3) P(A) (4) P(B)P(A∪B) P(A ∩ B) P(A ∪ B) P(A ∪ B)56. The period of sin 4 x+cos 4 x will be :(1) 3π (2) 2π (3) π (4) π2 2→→→→→→57. a x (b x c ) is equal to :→ → → → → →(1) (a . c) b - (a . b ) . c→ → → → → →(2) ( a . c) b + (a . b) . c→ → → → → →(3) (a . b) c + (a . b) . c→ → → → → →(4) (a . b) c - (a . c) . b58. The angle between the vectors (i+j) abd (j+k) is(1) π (2) 0 (3) π (4) π4 4 359. The area of the region bounded by the curves y = x sin x, axis of x, x= 0 andx = 2πππ will be :(1) 8π (2) 4π (3) 2 π (4) πππ π/260. log sin x dx is equal to :0b(1) π log 1 (2) π log 2 (3) π log 1 (4) π log 22 2 261. f(x) dx is equal toab a b(1) f (x-a-b) dx (2) f(a-x)dx (3) f(a+b-x)dx (4) noneof these


ππ π/2a 0 a62. sin 2x log tan x dx is equal to :0(1) 2π (2) π (3) 0 (4) π/2ππ63. cos 3 x dx is equal to :0(1) 4π (2) 2π (3) π (4) 0ππ π/264. 1 dx is equal to :01 + √√tan x65. cot x dx is equal to :(1) log tan x + C (2) log sec x + C(3) log cosec x + C (4) log sin x + C66. If z = x + y iy then |z – 5| is equal to :(1) √ (x – y) 2 + 5 2 (2) √ ( x – 5) 5 + y 2(3) √ x 2 + (y – 5) 2 (4) √ (x – 5) 2 + (y – 5) 267. If ααand ββare the roots of the equation 4x 2 + 3x + 7 = 0 then 1 + 1 isequal is :ααββ(1) 7 (2) 2 (3) -3 (4) 33 7 7 7⎯68. 2,357 is equal to :(1) 2379 (2) 2355 (3) 2355 (4) none of these999 999 99769. If the second term of a G.P. is 2 and the sum of its infinite terms is 8, thenits first therm is :(1) 2 (2) 4 (3) 6 (4) 870. (1+2+3+….+n) is equal to :(1) n(n + 1) 2 (2) n 2 (3) n(n + 1) (4) n(n-1)2 2 271. For n ∈∈N, 2 3 n - 7n – 1 is divisible by :(1) 50 (2) 49 (3) 51 (4) 4872. If x = 2 + 2 1/3 + 2 2/3 , then x 3 – 6x 2 + 6x is equal to :


(1) 0 (2) 1 (3) 2 (4) 373. If (1-x) n = C 0 + C 1 x + ….+ C n x n then C 1 + 2C 2 + 3C 3 +…..+ nC n is equal is:(1) n.2 n-1 (2) (n – 1) 2n-1 (3) (n + 1) 2n (4) 2 n-1 - 174. Determinate 1 + ib c + idc – id a – ib is equal to :(1) a 2 -b 2 +c 2 +d 2 (2) a 2 +b 2 -c 2 -d 2(3) (a 2 +b 2 ) (c 2 +d 2 ) (4) (a+b) (a-b)75. 43 1 635 7 4 is equal to:17 3 2(1) – 6 (2) – 110 (3) 0 (4) 15076. If A = 1 0 then A2 is equal to:0 1(1) 0 0 (2) 0 0 (3) 1 0 (4) 1 10 0 0 1 0 1 1 177. If A = 1 1 then A n is equal to :0 1(1) 1 n n (2) n n (3) 1 n (4) 1 10 1 0 n 0 1 0 178. If A and B are the invertible matrix of the required order then the valueof (AB) -1 will be :(1) [(AB)′] -1 (2) A -1 B -1 (3) B -1 A -1 (4) (BA) -179. The value of sin 3x is :(1) 4 sin x – 3 sin 3 x (2) 4 sin x + 3 sin 3 x(3) 3 sin x – 4 sin 3 x (4) 3 sin x + 4 sin 3 x80. The imaginary roots of (-1) 1/3 is :(1) 1 ± √ 3i (2) ± i (3) - 1 ± √3 (4) 1 ± √3i4 2 2sin iθθ81. The argument and modulus of the e θ is :(1) 1, sin hθ (2) 1, π/2 (3) e cos θ, sin hθ (4) e sin θ, sin hθ82. The minimum distance of a point (x, y) from a line ax + by + c = 0, is :


(1) |ax1 + by1 + c| (2) |ax1 + by1 + c|√ a 2 + b 2√a 2 + b 2 - c(3) |ax1 + by1 + c| (4) |ax1 + by1 + c|√ a 2 + b 2 + c 2√ a 2 + b 2 + c83. A straight line through ( 1, 1) and parallel to the line 2x + 3y – 7 = 0 is :(1) 2x + 3y + 5 = 0 (2) 3x – 2y + 7 = 0(3) 3x + 2y – 8 = 0 (4) 2x + 3y – 5 = 084. Equation of the straight line passing through the points (-1, 3) and (4, -2)is :(1) x- y = 3 (2) x + y = 3 (3) x – y = 2 (4) x + y = 285. The general equation of circle passing through the point of intersection ofcircle S = 0 and line P = 0, is :(1) S + λP = 0, λ ∈ R (2) 6S + 4P = 0(3) 3S + 4P = 0 (4) 4S + 5P = 086. The equation of the radial axis of two circle x 2 + y 2 + 2g 1 x + 2f 1 y + c 1 = 0and x 2 + y 2 + 2g 2 x + 2f 2 y + c 2 = 0, is :(1) 2 (g 1 – g 2 ) x + 2 (f 1 – f 2 ) y – c 1 – c 2 =0(2) 2 (g 2 – g 1 ) x + 2 (f 1 – f 2 ) y + c 1 – c 2 = 0(3) 2 (g 1 – g 2 ) x + 2 (f 1 – f 2 ) y + c 1 – c 2 = 0(4) 2 (g 1 – g 2 ) x + 2 (f 1 – f 2 ) y + c 2 – c 1 = 087. If f (x) = cos (log x), then f(x) f(y) – 1 [f(x) – f (xy) ] is equal to :y(1) 0 (2) f(x+y) (3) f(x) (4) f(xy)y88. If f(x) = x = y, then the value of f(y) is :x - 1(1) 1 –x (2) x + 1 (3) x – 1 (4) x89. lim 1 2 + 2 2 + 1___ is equal to :n →→ →∞∞13 + n3 23 + n3 2 n(1) 1 log 2 (2) 3 log 2 (3) 1 log 2 (4) 1 log 32 3 290. lim x2 – a2 is equal to :x →→ax - a(1) ∞ (2) 0 (3) a (4) 2a91. d (2 x ) is equal to :dx


(1) 1 (2) 2 x log 2 (3) x log 2 (4) 092. Differential coefficient of x 3 w.r.t. x 2 will be :(1) 3 (2) 2 (3) 3 x (4) 3x 22x 3x 2 293. d (tan x ) is equal to :dx(1) cosec 2 x (2) sec x tan x (3) cosec x cot x (4) sec 2 x94. The coordinates of the point where the tangent to the curve x2 + y2 – 2x –3 = 0 is parallel to the axis of x is :(1) 1. ± √3 (2) (1,0) (3) 1, ± 2 (4) (1. ±√ 2)95. The point at which tangent to the curve y = ττ2x at the point (0, 1) meetsthe x-axis is :(1) (1, 0) (2) (- ½, 0) (3) (2, 0) (4) (0, 2)96. Maximum value of slope of a tangent to the curve y = - x 3 + 3x 2 + 2x – 27will be :(1) 11 (2) – 4 (3) 5 (4) 297. m sin √√x dx is equal to :√√x(1) – 2 cos √ x + C (2) 2 cos √ x + C (3) 2 sin √ x + C (4) sin √x+C98. Correct statement is :(1) (AB) -1 = B -1 A -1 (2) (AB) -1 = A -1 B -1 (3) (AB) T = A T B T (4) (AB) -1 = A -1 B -199. If the matrix P = 1 2 and Q = - 1 0 then the correctstatement is : -3 0 2 3(1) P + Q = I (2) PQ ≠ QP (3) Q 2 = Q (4) P 2 = PANSWER SHEET1.(1) 2.(4) 3.(4) 4.(4) 5.(4) 6.(3) 7.(2) 8.(4) 9.(1) 10.(3) 11.(2)12.(2) 13.(4) 14.(2) 15.(4) 16.(4) 17.(2) 18.(1) 19.(3) 20.(1) 21.(4) 22.(2)23.(2) 24.(2) 25.(3) 26.(2) 27.(3) 28.(3) 29.(4) 30.(3) 31.(2) 32.(4) 33.(2)34.(4) 35.(2) 36.(2) 37.(3) 38.(1) 39.(2) 40.(3) 41.(3) 42.(4) 43.(2) 44.(4)45.(3) 46.(4) 47.(2) 48.(4) 49.(1) 50.(4) 51.(2) 52.(1) 53.(2) 54.(4) 55.(2)56.(4) 57.(1) 58.(4) 59.(2) 60.(3) 61.(3) 62.(3) 63.(4) 64.(4) 65.(4) 66.(2)67.(3) 68.(2) 69.(2) 70.(3) 71.(2) 72.(3) 73.(1) 74.(2) 75.(3) 76.(3) 77.(3)78.(3) 79.(3) 80.(4) 81.(1) 82.(1) 83.(4) 84.(4) 85.(1) 86.(3) 87.(4) 88.(4)89.(3) 90.(4) 91.(4) 92.(3) 93.(4) 94.(3) 95.(2) 96.(3) 97.(1) 98.(1) 99.(2)


<strong>Question</strong> Bank


1. The number of free electrons per 10 mm of an ordinary copper wire is 2 x 10 21 . The averagedrift speed of the electrons is 0.25 mm/s. The current flowing is:A. 0.8 A B. 8 A C. 80 A D. 5 A2. Which of the following cells is more likely to be damaged due to short circuiting?A. Daniel B. Dry C. Acid D. Fuel3. A gas expands from 5 litre to 105 litre at a constant pressure 100N/m 2 . The work done isA. 1 Joule B. 4 Joule C. 8 Joule D. 10 Joule4. The Helium nuclei can be formed fromA. Hydrogen nuclei by process of chain reactionB. Hydrogen nuclei through nuclear fissionC. Hydrogen nuclei through nuclear fusion D. None of these5. In the atom bomb dropped by Americans in 1945 on Nagasaki, Japan, the fissionable materialused wasA. Helium 4 B. Plutonium 239 C. Uranium 235 D. Uranium 2336. The engine of a truck moving a straight road delivers constant power. The distance travelledby the truck in time t is proportional toA. t B. t 2 C. √t D. t 3/27. The velocity of electron in ground state ofhydrogen atom isA. 2 x 10 5m/sB. 2 x 10 6m/sC. 2 x 10 7m/sD. 2 x 10 8m/s8. The radius of the first orbit of the electron in a hydrogen atom is 5.3 x 10 -11 m; then the radiusof the second orbit must beA. 15.9 x 10 -11 m B. 10.6 x 10 m C. 21.2 x 10 -11 m D. 42.4 x 10 -11 m9. A person pushes a rock of 10 10 Kg mass by applying a force of only 10N for just 4 seconds.The work done isA. 1000 Joule B. 0 J C. nearly zero D. positive10. One can take pictures of objects which are completely invisible to the eye using camera filmswhich are sensitive toA. ultra-violet rays B. sodium light C. visible light D. infra-red rays11. Light from a 100 watt filament bulb is passed through an evacuated glass tube containingsodium vapour at a high temperature. If the transmitted light is viewed through a spectrometer,we will observeA. D 1 and D 2 lines of sodium with goodintensityB. dark lines where D 1 and D 2 lines should havebeen observedC. continuous radiation from the bulb only D. the entire emission spectrum of sodium


12. Under the action of a constant force, aparticle is experiencing a constant acceleration.The power isA. zero B. positiveC. negative D. increasing uniformlywith time13. If in a plane convex lens the radius of curvature of the convex surface is 10 cm and the focallength of the lens is 30 cm, the refractive index of the material of the lens will beA. 1.5 B. 1.66 C. 1.33 D. 314. A plane convex lens has radius of curvature 30 cm. If the refractive index is 1.33, the focallength of lens isA. 10 cm B. 90 cm C. 30 cm D. 60 cm15. A beam of light is converging towards a point I on a screen. A plane parallel plate of glass(thickness in the direction of the beam = t, refractive index = µ ) is introduced in the path of thebeam. The convergence point is shifted byA. t (µ - 1) away B. t (1 + 1/µ ) away C. t (1 - 1/µ ) nearer D. t (1 + 1/µ ) nearer16 . In Young's double silt experiment the separation between the silts is halved and the distancebetween the silts and screen is doubled. The fringe width will beA. unchanged B. halved C. doubled D. quadrupled17. Wavelength of red light is λ r , violet rays is λ v and X -ray is λ x then the order ofwavelengths isA. λ x >λ v >λ r B. λ v >λ x >λ r C. λ r >λ x >λ v D. λ r >λ v >λ18. The amount of work done by the labourerwho carries n bricks, each of mass m, to the roofof a house whose height is h isA. n mgh B. mgh/n C. zero D. ghn/m19. In LCR circuit in the state of resonance, which of the following statements is correct ? (cosφ)=A. 0 B. 0.5 C. 1 D. None of these20. In LCR circuit, phase difference between voltage and current cannot beA. 80° B. 90° C. 145° D. 0°21. If speed is plotted along x-axis and Kinetic energy against y-axis, then the graph obtained hasa shape similar to that ofA. circle B. ellipse C. hyperbola D. parabola22. A magnetic needle lying parallel to a magnetic field requires w units of work to turn itthrough 60°. The torque needed to maintain the needle in this position will beA. (√ 3) w B. w


C. (√ 3w)/2 D. 2w23. A vertical straight conductor carries acurrent vertically upwards. A point p lies to theeast of it at a small distance and another point Qlies to west of it at the same distance. Themagnetic field at p isA. greater than at Q B. same as at QD. greater or less at QC. less than at Q depending upon thestrength of the current24. In a parallel arrangement if (R 1 > R 2 ), the power dissipated in resistance R 1 will beA. less than R 2 B. same as R 2 C. more than R 2 D. none of these25. For a fuse wire to be installed in the supply line in a house which one of the following isimmaterial ?A. the specific resistance of the material of theB. the diameter of the fuse wirefuse wireC. the length of the fuse wire D. none of these26. If V is voltage applied, E a is emf drop across the armature, the armature current of a d.c.motor I a is given byA. (V + E a )/R a B. E a /R a C. V- E a /R a D. V/R a27. The current of 2.0 amperes passes through a cell of e.m.f. 1.5 volts having internal resistanceof 0.15Ω . The potential difference measured in volts across both the terminals of the cell will beA. 1.35 B. 1.50 C. 1.00 D. 1.2028. In this circuit, current ratio i 1 /i 2 depends uponA. R 1 , R 2and RC. R 1 andR 2B. R, R 1 ,R 2 and ED. E and R29. A cell of emf E is connected across a resistance r. The potential difference between theterminals of the cell is found to be V. The internal resistance of the cell must beA. 2(E - V)V/r B. 2(E - V)r/E C. (E - V) r/V D. (E- V)/r30. Copper and germanium are both cooled to 70 K from room temperature, thenA. resistance of copper increases while that ofgermanium decreasesB. resistance of copper decreases while that ofgermanium increasesC. resistance of both decreases D. resistance of both increases31. The potential difference between the points A and B of the electrical circuit given isA. 1.5 V B. 1.0 V


32. A moving coil galvanometer has a resistanceof 9.8Ω and gives a full scale deflection when acurrent of 10 mA passes tbrough it. The value ofthe shunt required to convert it into a miniammeter to measure current upto 500 mA isA. 0.02Ω B. 0.2Ω C. 2Ω D. 0.4Ω33. The total electrical resistance between the points A and B of the circuit shown in the figure isA. 9.02 Ω A. 15 ΩC. 30 Ω D. 100 Ω34. If the plates of a charged parallel plate capacitor are pulled away from each otherA. capacitanceB. energy increases C. voltage increases D. voltage decreasesincreases35. A parallel plate capacitor is charged by connecting its plates to the terminals of a battery. Thebattery remains connected and a glass plate is interposed between the plates of the capacitor,thenA. the charge on plates will be reducedB. the charge on plates will increaseC. the potential difference between the plates of the capacitor will be reducedD. the potential difference between the plates of the capacitor will increase36. A person weighing 70Kg wt lifts a mass of 30 Kg to the roof of a building 10 m high. If hetakes 50 sec to do so,then the power spent isA. 19.6 W B. 196 W C. 300 W D. 50 W37. Work done in carrying a charge q from A to B along a semi-circle isA. 2πrq B. 4πrqC. πrq D. 038. A particle A has charge +q and particle B has charge +4q with each of them having the samemass m. When allowed to fall from rest through same electrical potential difference, the ratio oftheir speed V A : V B will becomeA. 2:1 B. 1:2 C. 1:4 D. 4:139. The electric field at a small distance R from an infinitely long plane sheet is directlyproportional toA. R 2 /2 B. R/2 C. R -2 D. none of these40. In the diagram, the electric field intensity will be zero at a distanceA. between -q and +2q charge B. towards +2q on the line drawn


C. away from the line towardsD. away from the line towards -q+2q41. Wein's displacement law is given byA. λ m = B. T/λ m = C. λ m T = D. T = λ mconstant constant constant = constant42. If two electrons are forced to come closer to each to each other, then the potential energyA. becomes zero B. increases C. decreases D. becomes infinite43. The specific heat at constant pressure is greater than that of the same gas at constant volumebecauseA. at constant volume work is done in expanding the gasB. at constant pressure work is done in expanding the gasC. the molecular attraction increases more at constant pressureD. the molecular vibration increases more at constant pressure44. The specific heats of CO 2 at constant pressure and constant volume are 0.833 J/kg.K and0.641 J/kg.K respectively. If molecular weight of CO 2 is 44, what is the universal constant R?A. 4.19 x 10 7 erg/cal B. 848.8 J/gm/K C. 8.448 J/mol/K D. 4.19 J/cal45. The freezing point of the liquids decreases when pressure is increased, if the liquidA. expands while freezing B. contracts while freezingC. does not change in volume while freezing D. none46. The equation of a transverse wave on astretched string is given byy = 0.05 sin π (2t/0.002 -x/0.1 ) where x and yare expressed in metres and t in sec.The speed of the wave isA.100m/secB. 50 m/s C. 200 m/s D. 400 m/s47. The ratio of velocity of the body to the velocity of sound is calledA. Magic number B. Laplace number C. Natural number D. Mach number48. Television signals on earth cannot be received at distances greater than 100 km from thetransmission station. The reason behind this is thatA. the receiver antenna is unable to detect the signal at a distance greater than 100 kmB. the TV programme consists of both audio and video signalsC. the TV signals are less powerful than radio signalsD. the surface of earth is curved like a sphere49. A ball is thrown from a height of h m with an initial downward velocity v 0 . It hits the ground,loses half of its Kinetic energy & bounces back to the same height. The value of v 0 isA. √2gh B. √gh C. √3gh D. √2.5gh


50. A thick rope of rubber of density 1.5 x 10 3kg/m 3 and Young's modulus 5 x 10 6 N/m 2 , 8m inlength, when hung from ceiling of a room, theincrease in length due to its own weight isA. 9.6 x 10 - B. 19.2 x C. 9.6cm D. 9.6mm3 m 10 -5 m51. Water is falling on the blades of a turbine at a rate 6000Kg/min. The height of the fallis100m. What is the power gained by the turbine?A. 10KW B. 6KW C. 100KW D. 600KW52. If momentum of alpha-particle, neutron, proton, and electron are the same, the minimumK.E. is that ofA. alpha-particle B. neutron C. proton D. electron53. An electric motor while lifting a given load produces a tension of 4500 N in the cableattached to the load. If the motor winds the cable at the rate of 2m/s, then power must beA. 9 kW B. 15 kW C. 225 kW D. 9000 H.P54. If an electric iron electrons are accelerated through a potential difference of V volts. Takingelectronic charge and mass to be respectively e and m, the maximum velocity attained by theelectrons isA. 2eV/√m B. √(2eV)/m C. 2m/eV D. v 2 /8em55. A particle is moving on a circular track of radius 20 cm with a constant speed of 6 m/s. Itsacceleration isA. 0 B. 180 m/s 2 C. 1.2 m/s 2 D. 36 m/s 256. A satellite of the earth is revolving in a circular orbitwith a uniform speed v. If gravitational force suddenlydisappears, the satellite will:A. continue to move with the speed v along the original orbitB. move with the velocity v tangentially to the original orbitC. fall downward with increasing velocityD. ultimately come to rest somewhere on the original orbit57. The kinetic energy K of a particle moving along a circle of radius R depends on the distancecovered s as K = as 2 . The force acting on the part1cle isA. 2as 2 /R B. 2as(1 + s 2 /R) 1/2 C. as(1 + s 2 /R 2 ) 1/2 D. None of these58. Einstein was awarded Nobel Prize for his work inA. Photoelectric effect B. Special theory of relativityC. General theory of relativity D. None of these59. One second is defined to be equal toA. 1650763.73 periods of the Krypton clock B. 652189.63 periods of the Krypton clockC. 1650763.73 periods of the Cesium clock D. 9192631770 periods of the Cesium clock


60. The dimensions of energy and torque respectively areA. ML 2 T -2 and ML 2 T -2 B. MLT 2 and ML 2 T -2 C. ML 2 T -2 and MLT -2 D. MLT -2 and MLT -261. When Benzene diazonium chloride reacts with hypophosphorous acid, it producesA. benzene B. phenol C. phenylphosphite D. phenylphosphate62. The reaction of aliphatic primary amine with nitrous acid in cold producesA. nitrile B. alcohol C. diazonium salt D. secondary amine63. Ethylamine can be prepared by the action of bromine and caustic potash onA. acetamide B. propionamide C. formamide D. methyl cyanide64. The aldol condensation of acetaldehyde results in the formation ofA. CH 3 COCHOHCH 3 B. CH 3 CHOHCH 2 CHO C. CH 3 CH 2 CHOHCHO D. CH 3 CH 2 OH +CH 3 COOH65. Which compound reacts fastest with Lucas reagent at room temperature?D. 2-Methyl propan-2-A. Butan-l-ol B. Butan-2-ol C. 2-Methyl propan-l-olol66. The reaction with D 2 O, (CH 3 ) 3 CMgCl producesA. (CH 3 ) 3 CD B. (CH 3 ) 3 CO C. (CD 3 ) 3 CD D. (CD 3 ) 3 COD67. The reaction with alcoholic potash, l-chlorobutane givesA. 1-Butene B. 1-Butanol C. 2-Butene D. 2-Butanol68. The active nitrating agent during nitration ofbenzene is---A. NO 3 B. HNO 2 C. NO 2 D. HNO 369. The number of sigma and pi bonds in 1-buten-3-yne areA. 5 sigma and 5 pi B. 7 sigma and 3 pi C. 8 sigma and 2 pi D. 6 sigma and 4 pi70. The most stable carbonium ion among the cations isA. sec-butyl B. ter-butyl C. n-butyl D. none of these71. How many optically active stereo-isomers are possible for butane-2, 3-diol?A. 1 B. 2 C. 3 D. 472. B.P. and M.P. of inert gases areA. high B. low C. very high D. very low73. [CO(NH 3 ) 5 Br] SO 4 and [CO(NH 3 ) 5 SO 4 ] Br are examples of which type of isomerism ?A. Linkage B. Geometrical C. Ionization D. Optical74. The valency of Cr in the complex [Cr(H 2 O) 4 Cl 2 ] + isA. 3 B. 1 C. 6 D. 5


75. In Nessler's reagent, the ion isA. Hg + B. Hg 2+ C. HgI 22 -D. HgI 42 -76. In solid CuSO 4 .5H 2 O, copper is co-ordinated toA. five water molecules B. four water molecules C. one sulphate ion D. one water molecule77. Which of the following is a weak acid?A. HCl B. HBr C. HP D. HI78. When SO 2 is passed through acidified K 2 Cr 2 O 7 solution,A. the solution turns blue B. the solution is decolourisedC. SO 2 is reduced D. green Cr 2 (SO 4 ) 3 is formed79. Which of the following has lowest boiling point?A. H 2 O B. H 2 S C. H 2 Se D. H 2 Te80. Nitric oxide is prepared by the action of dil. HNO 3 onA. Fe B. Cu C. Zn D. Sn81. The laughing gas isA. nitrousoxideB. nitricoxideC. nitrogentrioxideD. nitrogenpentaoxide82. Ordinary glass isA. sodium silicate B. calcium silicateC. calcium and Sodium silicate D. copper silicate83. The chemical name of phosgene isA. Phosphene B. Carbonyl chlorideC. PhosphorousoxychlorideD. Phosphoroustrichloride84. Which one of the following is strongest Lewis acid?A. BF 3 B. BCl 3 C. BBr 3 D. BI 385. Three centred bond is present inA. NH 3 B. B 2 H 6 C. BCl 3 D. AlCl 386. Plaster of Paris isA. CaSO 4 .H 2 O B. CaSO 4 .2H 2 O C. CaSO 4 .1/2 H 2 O D. CaSO 4 .3/2 H 2 O87. Rocky impurities present in a mineral arecalledA. flux B. gangue C. matte D. slag88. Free hydrogen is found inA. acids B. water C. marsh gas D. water gas89. When zeolite, which is hydrated sodium aluminium silicate, is treated with hard water; the


sodium ions are exchanged withA. H + B. K + 2-C. SO 4 D. Mg 2+90. On passing 0.3 faraday of electricity through aluminium chloride, the amount of aluminiummetal deposited on cathode is (Al = 27)A. 0.27 g B. 0.3 g C. 2.7 g D. 0.9 g91. The migration of colloidal particles under influence of an electric field is known asA. Electro-osmosis B. Brownian movement C. Cataphoresis D. Dialysis92. In a colloidal state, particle size ranges fromA. 1 to 10 A o B. 20 to 50 A o C. 10 to 1000 A o D. 1 to 280 A o93. The half-life of a first order reaction is 69.35. The value of rate constant of the reaction isA. 1.05 -1 B. 0.15 -1 C. 0.015 -1 D. 0.0015 -194. Heat of neutralisation of a strong acid andstrong base is alwaysA. 13.7 B. 9.6Kcal/mol Kcal/molC. 6Kcal/molD. 11.4Kcal/mol95. In exothermic reactions,A. H R =H P B. H R >H P C. H R < H P D. None of the above96. Which is a buffer solution?A. CH 3 COOH + B. CH 3 COOH + C. CH 3 COOH + NH 4 Cl D. NaOH + NaClCH 3 COONa CH 3 COONH 497. The pH of 0.01 M solution of HCl isA. 1.0 B. 2.0 C. 10.0 D. 11.098. In which of the following case does the reaction go fastest to completion?A. k = 10 2 B. k = 10 -2 C. k = 10 D. k = 199. What quantity of limestone (CaCO 3 ) on heating will give 28 kg of CaO?A. 1000 kg B. 56 kg C. 44 kg D. 50 kg100. The percentage of oxygen in NaOH isA. 40 B. 16 C. 18 D. 10101. If we take 44 g of CO 2 and 14 g of N 2 ,what will be the mole fraction of CO 2 in themixture?A. 1/5 B. 1/3 C. 1/2 D. 1/4102. The molarity of a solution of Na 2 CO 3 having 5.3 g/250 ml of solution isA. 0.2 M B. 2 M C. 20 M D. 0.02 M103. A gas is initially at 1 atm pressure. To compress it to 1/2th of its initial volume, pressure tobe applied is


A. 1 atm B. 4 atm C. 2 atm D. 1/4 atm104. The value of R in calorie/degree/mole isA. 0.0831 B. 8.31 C. 8.31 x 10 7 D. 1.987105. Which of the following possesses zero resistance at 0 K?A. Conductors B. Semi-conductors C. Super-conductors D. Insulators106. CsCl has lattice of the typeA. ccp B. fcc C. bcc D. hcp107. In the reaction between sodium and chlorine to form sodium chloride,A. sodium atom isreducedB. sodium ion isreducedC. chlorine atom isreducedD. chloride ion isreduced108. Octahedral molecular shape exists in______ hybridisation.A. sp 3 d B. sp 3 d 2 C. sp 3 d 3 D. sp 2 d 2109. NH 3 and BF 3 form an adduct readily because they formA. a co-ordinate bond B. a covalent bond C. an ionic bond D. a hydrogen bond110. Diagonal relationship exists betweenA. Li and Mg B. Na and Mg C. K and Mg D. Al and Mg111. Which element has the highest electro-negativity?A. F B. He C. Ne D. Na112. Loss of a -particle is equivalent toA. loss of two neutrons only B. loss of two protons onlyC. loss of two neutrons and loss of two protons D. none of the above113. Stable compounds in + 1 oxidation state are formed byA. B B. Al C. Ga D. Th114. Sodium hexametaphosphate is used asA. a cleansing agent B. an insecticide C. a water softner115. The strongest acid isA.ClO 3 (OH)B.ClO 2 (OH)C.SO(OH) 2D.SO 2 (OH) 2D. an iron exchangeresin116. Which one among the following pairs of ions cannot be separated by H 2 S in dilutehydrochloric acid?A. Bi 3+ , Sn 4+ B. Al 3+ , Hg 2+ C. Zn 2+ , Cu 2+ D. Ni 2+ , Cu 2+117. The alkane would have only the primary and tertiary carbon is


A. Pentane B. 2-methylbutaneC. 2, 2-dimethylpropaneD. 2, 3-dimethylbutane118. The product of reaction of alcoholic silver nitrite with ethy1 bromide isA. ethane B. ethene C. nitroethane D. ethyl a1coho1119. Formy1 chloride has not been so prepared. Which one of the following can function asformyl chloride in formulation?A. HCHO + HCl B. HCOOCH 3 + HCl C. CO + HCl D. HCONH 2 + HCl120. Amongst the following, the most basic compound isA. Benzylarnine B. Aniline C. Acetanilide D. p-Nitroaniline121. If the roots of x 2 - bx + c = 0 areconsecutive integers, then b 2 - 4c is equal toA. 4 B. 3 C. 2 D. 1122. Condition that the two lines represented by the equation ax 2 + 2hxy + by 2 = 0 to theperpendicular isA. a = - b B. ab = 1 C. a = b D. ab = -1123. If A ⊆ B, then A ∩ B is equal toA. B c B. A c C. B D. A124. In order that the function f(x) = (x + 1) cot x is continuous at x = 0, f(0) must be defined asA. f(0) = 0 B. f(0) = e C. f(0) = 1/e D. none of the above125. The eccentricity of the ellipse 16x 2 + 7y 2 = 112 isA. 4/3 B. 7/16 C. 3/√7 D. 3/4126. If z 1 , z 2 , z 3 are three complex numbers in A.P., then they lie onA. a circle B. an ellipse C. a straight line D. a parabola127. If [(a 2 + 1) 2 ]/(2a - i) = x + iy, then x 2 + y 2 isequal toA. [(a 2 +1) 4 ]/(4a 2 +1)B. [(a +1) 2 ]/(4a 2 +1)C. [(a 2 -1) 2 ]/(4a 2 D. none of-1) 2 the above128. The vertices of a triangle are (0, 0), (3, 0) and (0, 4). Its orthocentre is atA. (3/2, 2) B. (0, 0) C. (1, 4/3) D. none of the above129. The eccentricity of the conic 9x 2 - 16y 2 = 144 isA. 5/4 B. 4/3 C. 4/5 D. √7130. The vertices of a triangle are (0, 3), (-3, 0) and (3, 0). The co-ordinates of its orthocentre are


A. (0, 2) B. (0, -3) C. (0, 3) D. (0, -2)131. If t is the parameter for one end of a focal chord of the parabola y 2 = 4ax, then its length isA. a [t - (1/t)] B. a [t + (1/t)] C. a [t - (1/t)] 2 D. a [t + (1/t)] 2132. The value of cos 2 θ + sec 2 θ is alwaysA. equal to 1 B. less than 1C. greater than or equal to 2 D. greater than 1, but less than 2133. The number of points of intersection of 2y= 1 and y = sin x, -2π ≤ x ≤ 2π isA. 2 B. 3 C. 4 D. 1134. If sin θ 1 + sin θ 2 + sin θ 3 = 3, then cos θ 1 + cos θ 2 + cos θ 3 =A. 0 B. 1 C. 2 D. 3135. The number of solutions in 0 ≤ x ≤ π/2 of the equation cos 3x tan 5x = sin 7x isA. 5 B. 7 C. 6 D. none of the above136. One end of a diameter of the circle x 2 + y 2 - 4x - 2y - 4 = 0 is (5, -6), the other end isA. (4, -9) B. (-9, -4) C. (4, 9) D. (9, -4)137. The set of values of m for which both the roots of the equation x 2 - (m + 1)x + m + 4 = 0 arereal and negative consists of all m, such thatA. -3 ≥ m or m ≥ 5 B. -3 < m ≤ 5 C. - 4 < m ≤ -3 D. -3 < m ≤ -1138. Let P n (x) = 1 + 2x + 3x 2 + ...... + (n + 1) x n be a polynomial such that n is even. Then thenumber of real roots of P(x) = 0 isA. 1 B. n C. 0 D. none of the above139. The next term of the sequence 1, 3, 6, 10,........ isA. 16 B. 13 C. 15 D. 14140. If H is the harmonic mean between P and Q, then H/P + H/Q isA. (P + Q)/PQ B. PQ/(P + Q) C. 2 D. none of the above141. A class is composed of two brothers and six other boys. In how many ways can all the boysbe seated at a round table so that the two brothers are not seated besides each other?A. 4320 B. 3600 C. 720 D. 1440142. The binomial coefficient of the 4th term in the expansion of (x - q) 5 isA. 15 B. 20 C. 10 D. 5143. For x ≠ 0, the term independent of x in the expansion of (x - x -1 ) is equal to


A. 2n C n B. [(-1) n ] [ 2n C n ] C. [(-1) n ] [ 2n C n + 1 ] D. 2n C n + 1144.ka 1 b 1 c 1a 2 b 2 c 2a 3 b 3 c 3isequaltoa 1b 1kc 1A.a 2kb 2c 2ka 3b 3c 3ka 1kb 1kc 1B.ka 2kb 2kc 2ka 3kb 3kc 3ka 1b 1c 1C.ka 2b 2c 2ka 3b 3c 3


ka 1b 1c 1D.a 2kb 2c 2a 3b 3kc 3145. Oneroot of theequation3x -83333x -83333x -8= 0 is whichof thefollowing?A. 2/3 B. 8/3 C. 16/3 D. 1/3abcq-by146. If | A | =xpyqzrand | B | =-pra-c-xz, thenA. | A | = 2 | B | B. | A | = | B | C. | A | = - | B | D. none of the above147. Equation of the sphere with centre (1, -1, 1) and radius equal to that of sphere 2x 2 + 2y 2 +2z 2 - 2x + 4y - 6z = 1 isA. x 2 + y 2 + z 2 - 2x + 2y - 2z + 1 = 0 B. x 2 + y 2 + z 2 + 2x - 2y + 2z + 1 = 0C. x 2 + y 2 + z 2 - 2x + 2y - 2z - 1 = 0 D. none of the above148. Equation of the line passing through thepoint (1, 1, 1) and parallel to the plane 2x + 3y +3z + 5 = 0 isA. (x - 1)/1 = (y - 1)/2 = B. (x - 1)/-1 = (y - 1)/1(z - 1)/1C. (x - 1)/3 = (y - 1)/2 =(z - 1)/1= (z - 1)/-1D. (x - 1)/2 = (y - 1)/3 =(z - 1)/1


149. If a, b, c are constants such that a and c are of opposite signs and r is the correlationcoefficient between x and y, then the correlation coefficient between ax + b and cy + d isA. (a/c)r B. r C. - r D. (c/a)r150. From a deck of 52 cards, the probability of drawing a court card isA. 3/13 B. 1/4 C. 4/13 D. 1/13151. A binomial probability distribution is symmetrical if p, the probability of success in a singletrial, isA. > 1/2 B. < 1/2 C. < q, where q = 1 - p D. = 1/2152. The binomial distribution whose mean is 10 and S.D. is 2√2 isA. (4/5 + 1/5) 50 B. (4/5 + 1/5) 1/50 C. (4/5 + 5/1) 50 D. none of the above153. tan (cot -1 x) is equal toA. π/4 - x B. cot (tan -1 x) C. tan x D. none of the above154. If f(x) is an odd periodic function withperiod 2, then f(4) equalsA. - 4 B. 4 C. 2 D. 0155. The function f(x) = [(x 3 + x 2 - 16x + 20)]/(x - 2) is not defined for x = 2. In order to makef(x) continuous at x = 2, f(2) should be defined asA. 0 B. 1 C. 2 D. 3156. Let f and g be differentiable functions satisfying g'(a) = 2, g(a) = b, and fog = 1 (identityfunction). Then f'(b) is equal toA. 0 B. 2/3 C. 1/2 D. none of the above157. A cone of maximum volume is inscribed in a given sphere. Then the ratio of the height ofthe cone to the diameter of the sphere isA. 3/4 B. 1/3 C. 1/4 D. 2/3158. The function is decreasing in the intervalA. - ∞ < x < -10/3 B. 0 < x < ∞ C. -3 < x < 3 D. -10/3 < x < 0159. Suppose that f''(x) iscontinuous for all x and tf'(t) dt = 0,f(0) = f'(1). Ifthen the value of f(1) isA. 3 B. 2 C. 9/2D. none ofthe above160. Integrating factor of differential equation cos x (dy/dx) + y sin x = 1 isA. sin x B. sec x C. tan x D. cos x


161. Ifdx/(1 + 4x 2 ) =π/8,then the value of a isA. π/2 B. 1/2 C. π/4 D. 1162. The maximum value of (log x)/x isA. 2/e B. 1/e C. 1 D. e163. If one root of the equation x 2 + px + 12 = 0is 4, while the equation x 2 + px + q = 0 hasequal roots, then the value of q isD. none ofA. 49/4 B. 4/49 C. 4the above164. The sum of the series 1/2 + 1/3 + 1/6 + ....... to 9 terms isA. -5/6 B. -1/2 C. 1 D. -3/2165. The sum of all two digit numbers, which are odd isA. 2475 B. 2530 C. 4905 D. 5049166. How many ten digit numbers can be formed by using the digits 3 and 7 only?A. 10 C 1 + 9 C 2 B. 2 10 C. 10 C 2 D. 10!167. If x and y are real and different and u = x 2 + 4y 2 + 9z 2 - 6xyz - 3zx - 2xy, then u is alwaysA. non-negative B. zero C. non-positive D. none of the above168. If a be a non-zero vector, then which of the following is correct?A. a . a = 0 B. a . a > 0 C. a . a ≥ 0 D. a . a ≤ 0169. If two vectors a and b are parallel and haveequal magnitudes, thenA. they are equal B. they are not equalC. they may or may notbe equalD. they do not have thesame direction170. In a triangle, the lengths of the two larger sides are 10 and 9 respectively. If the angles arein A.P., then the length of the third side can beA. 5 ± √6 B. 3√3 C. 5 D. none of the above171. The three lines 3x + 4y + 6 = 0, √2x + √3y + 2√2 = 0, and 4x + 7y + 8 = 0 areA. sides of a triangle B. concurrent C. parallel D. none of the above172. The pole of the straight line 9x + y - 28 = 0 with respect to the circle 2x 2 + 2y 2 - 3x + 5y - 7= 0 isA. (3, 1) B. (1, 3) C. (3, -1) D. (-3, 1)


173. If the sets A and B are defined as A = { (x, y) : y = e x , x ∈ R }, B = { (x, y) : y = x, x ∈ R },thenA. A ∪ B = A B. A ∩ B = φ C. A ⊆ B D. B ⊆ A174. Thevalue of theintegralA. a B. 2a C. 3a{ f(x)/[f(x) + f(2a- x)] }dx is equaltoD. none ofthe above175. The slope of the normal at the point (at 2 , 2at) of the parabola y 2 = 4ax isA. 1/t B. t C. - t D. -1/t176. If z is any complex number such that | z + 4 | ≤ 3, then the greatest value of | z + 1 | isA. 2 B. 6 C. 0 D. - 6177. The equation cos x + sin x = 2 hasA. only one solution B. two solutionsC. no solution D. infinite number of solutions178. The most general value of θ, which satisfies both the equations tan θ = -1 and cos θ = 1/√2will beA. nπ + (7π/4) B. nπ + (-1) n (7π/4) C. 2nπ + (7π/4) D. none of the above179. A spherical ball of radius r placed on theground subtends an angle of 60 o at a point A ofthe ground. Then the distance of the point Afrom the centre of the ball isD. none ofA. 3r B. 2r C. 4rthe above180. In a triangle ABC, a 2 cos 2B + b 2 cos 2A + 2ab cos (A - B) is equal toA. c B. c 2 C. 2c D. none of the above


1. The radius of curvature of a spherical surface is measured usingA. a spherometer B. spectrometer C. screw gauge D. slide callipers2. If the dimensions of length are expressed as G x , C y , h z , where G, C, h are universalgravitational constant, speed of light and Plank's constant respectively, thenA. x = 1/2, y = 1/2 B. x = 1/2, z = 1/2 C. y = 1/2, z = 3/2 D. y = + 3/2, z = 1/23. The dimensional formula of electric field strength is:A. MLT -2 I -1 B. MLT -3 A -1 C. T -2 A -1 D. MLTA -24. A man throws a ball in air in such a way that when the ball is in its maximum height he throwsanother ball. If the balls are thrown after the time difference of 1 sec, then what wilt be the heightattained by themA. 19.6 m B. 9.8 m C. 4.9 m D. 2.45 m5. If the velocity time graph of a body is a straight line sloping downwards, the body hasA. acceleration B. declaration C. zero acceleration D. constant acceleration6. Which one of the following equations represents the motion of body with finite constantacceleration?A. y = at B. y = at + bt 2 C. y = at + bt 2 + ct 3 D. y = at + bt7. What is the magnitude of the velocity of thebody when it is projected horizontally from apoint above the ground after 0.2 seconds?A. √2 ms -1 B. 2√2 ms -1 C. 3√2 ms -1 D. 4√2 ms -18. A string can withstand a tension of 25 N. What is the greatest speed at which a body of mass 1kg can be whirled in a horizontal circle using 1 m length of the string?A. 25 ms -1 B. 5 ms -1 C. 75 ms -1 D. 10 ms -19. An object tied to a piece of string is whirled in a vertical circle, at constant speed. The tentionin the string is maximum atA. A B. BC. C D. D10. The maximum force of friction that comes into play is calledA. limiting friction B. kinetic friction C. static friction D. minimum friction11. A body of mass 5 Kg is raised vertically to aheight of 10 m by a force of 170 N. The finalvelocity of the body isA. 15 ms -1 B. 17 ms -1 C. 20 ms -1 D. 22 ms -1


12. A cyclist moving at a speed of 17.64 km/h describes a circle of radius 9.8 m. If the cyclist isheld in balance, the co-efficient of friction between the tyre and the ground isA. 0.25 B. 0.29 C. 0.36 D. 0.3513. Two bodies with masses m 1 and m 2 have equal kinectic energies. If P 1 and P 2 are theirrespective momenta, then P 1 = P 2 isA. m 1 : m 2 B. m 2 : m 1 C. m 2 21 : m 2 D. √m 1 : √m 214. In elastic collision,A. only energy is conserved B. only momentum is conservedC. both energy and momentum is conserved D. none of these15. The velocity of a particle whose kineticenergy is equal to the rest energy isA. (1/2) C B. C C. √3/3 D. √3 C16. The propeller of a ship makes 350 rev. while its speed increases from 200 rpm to 500 rpm.Then the time taken for this isA. 1 min B. 1.2 minute C. 5.3 seconds D. 53 seconds17. The K.E. needed to project a body from the earth's surface to infinity isA. mgR B. 2 mgR C. 1/2 (mgR) D. 1/4 (mgR)18. The distance of two planets from the sun are 10 13 and 10 12 meters respectively. The ratio oftime period of these two planets isA. √10 B. 1/√10 C. 100 D. 10√1019. Poisson ratio is the ratio ofA. the linear strain to the lateral strain B. the lateral strain to the linear strainC. the linear stress to the lateral stress D. the lateral stress to the linear stress20. Two wires L and M are of the same materialand of the same length, but the diameter of L istwice that of M stretching force applied to L isfour times that of M. Then the ratio of theelongation of L to that of M isA. 1 : 4 B. 4 : 1 C. 1 : 1 D. 2 : 121. Which of the substance breaks just beyond the elastic limit?A. Elastic B. Malleable C. Brittle D. Ductile22. A stone of mass 16 kg is attached to a string 144-meter-long and is whirled in a horizontalcircle. The maximum tension the string can stand is 16 N. The maximum velocity of revolutionthat can be given to the stone without breaking it will beA. 12 ms -1 B. 14 ms -1


C. 16 ms -1 D. 20 ms -123. A vessel containing 0.1 m 3 of air at 76 cm of Hg pressure is connected to an evacuated vesselof capacity 0.09 m 3 . The resultant air pressure isA. 20 cm of Hg B. 30 cm of Hg C. 40 cm of Hg D. 50 cm of Hg24. Two gases A and B having the same temperature T, same pressure P and the same volume Vare mixed. If the mixture is at the same temperature T and occupies a volume V, the pressure ofthe mixture isA. P B. 2P C. P/2 D. 4P25. A solid ball of metal has spherical cavityinside it. If the ball is heated, the volume of thecavity willA. increase B. decreaseC. remainthe sameD.disappear26. If the law of heat conduction is written in the form of Ohm's law, then the quantity similar toelectrical resistance isA. A/dλ B. Ad/λ C. Aλ/d D. d/Aλ27. The work done from 250 cals of heat isA. 1045 ergs B. 1045 joules C. 1045 watt D. 1045 N28. The time taken by a particle executing S.H.M of period T to move the mean position to halfthe maximum displacement isA. T/2 B. T/4 C. T/8 D. T/1229. Let g be the acceleration due to gravity atearth's surface and K be the rotational K.E. ofthe earth. Suppose the earth's radius decreasesby 2%, thenA. g decreases by 2% B. g decreases by 4%and K decreases by 4%C. g increases by 4%and K decreases by 4%and K increases by 2%D. decreases by 4% andK increases by 4%30. A particle of mass m is hanging vertically by an ideal spring of force constant K. If the massis made to oscillate vertically, its total energy isA. maximum at the extreme position B. maximum at the equilibriumC. minimum at the equilibrium D. same at all position31. Velocity of sound in CO 2 is less than in hydrogen becauseA. CO 2 is heavier than hydrogenB. CO 2 is a compound and hydrogen is anelementC. CO 2 is more soluble in water D. CO 2 can be more easily liquefied


32. The velocity of sound in air at room temperature is 110 m/sec. The length of the wavecoming from a vibrating fork at frequency 275 isA. 0.4 m B. 100 m C. 825 m D. 1375 m33. The temperature at which velocity of sound in air is double its velocity at 0°C isA. 435°C B. 694°C C. 781°C D. 819°C34. Static electricity is produced byA. induction B. frictionC. both induction and D. none of the abovefriction35. Surface charge density on a pear shaped conductor isA. maximum in the middle position B. maximum near the tapering endC. maximum near the broad end D. equal throughout the surface36. A given charge situated at a certain distance from an electric dipole in the end on positionexperiences a force F. If the distance of the charge is doubled, the force acting on the charge willbeA. 2F B. F/2 C. F/4 D. F/837. A piece of fuse wire melts when the current is 5 A. The energy produced then is 1 J/s. Theresistance of the fuse in ohm isA. 0.04 B. 0.1 C. 0.5 D. 1038. The gravitational force between two point masses m 1 and m 2 at separation r is given byF = (m 1 m 2 )/r 2 Then constant KA. depends on systems of units only B. depends on medium between masses onlyC. depends of both masses and units D. none of these39. A piece of copper and another of germaniumare cooled from room temperature to 80 K. Theresistance ofA. each of themincreasesB. each of themdecreasesC. copper increases and D. germanium increasesgermanium decreases and copper decreases40. In a given thermocouple, the temperature of the cold junction is 20°C, while the neutraltemperature is 27°C. What will be the temperature of immersion ?A. 420°C B. 425°C C. 520°C D. 525°C41 When different parts of a metal are kept at different temperature and current is passed throughit, heat is either evolved or absorbed. The effect is calledA. Peltier effect B. Seebeck effect C. Thompson effect D. Joule effect42. A storage battery is to be charged from a d.c. supply which terminal of the battery beconnected to the positive side of the lineA. positive B. negative


C. both positive and negative D. first negative and after the lapse of 5 minutespositive43. The force between two parallel wires carrying currents in the same direction is aA. force of attraction B. force of repulsionC. no resultant force between the wires44. The motion of an electric charge producesA. only an electric field B. only a magnetic fieldC. both magnetic andelectric fieldD. none of the aboveD. resultant force acting perpendicular to theflow of wires45. An ammeter is connected in series with a 2V circuit containing a 2V battery when the switchis closed, the ammeter shows high deflection and comes to zero. The circuit may contain aA. resistance of 20Ω B. fuse C. diode D. triode46. Ferromagnetic substances haveA. very high permeability and susceptibility B. low permeability but high susceptibilityC. high permeability and low susceptibility D. none of these47. The permeability of the paramagnetic substance isA. very large B. very small C. negative48. When a material is subjected to a small fieldH, the intensity of magnetisation is proportionaltoA. √H B. H C. H 2 D. 1/√HD. small but more than149. In a capacitance circuit the resistance isA. ω C B. 1/ω C C. 1/√ω C D √ω x C50. In electromagnetic induction, the induced e.m.f. is independent ofA. change of flux B. timeC. number of lines of force D. resistance of the cells51. A coil of area A is kept perpendicular to a magnetic field B. If coil is rotated by 180 0 , thenchange in the flux will beA. BA B. zero C. 2BA D. 3BA52. The displacement current flows in the dielectric of a capacitor when the P.D. across its platesA. is increasing with time B. is not decreasing with timeC. has assured a constant value D. becomes zero53. Electromagnetic wavesA. are longitudinal B. travel in free space atwavesthe speed of lightC. are produced by D. travel with the same


charges moving withuniform velocityspeed in all media54. The frequency of visible light is of the order ofA. 10 8 Hz B. 10 18 Hz C. 10 15 Hz D. 10 12 Hz55. A concave mirror of focal length 15cm forms an image at a distance of 40 cm from it. Thedistance of the object from the mirror isA. 10 cm B. 20 cm C. 24 cm D. 30 cm56. Binoculars are made conveniently short by making use of right angled isosceles prism ofglass. In a normal pair of binoculars, the number of prism isA. 1 B. 2 C. 4 D. 557. A ray incident on a 60° prism of refractiveindex √ 2 suffers minimum deviation. The angleof incidence isA. 0° B. 45° C. 60° D. 75°58. Two electron beams having velocities in the ratio of 1 : 2 are subjected separately to identicalmagnetic field. The ratio of deflection produced isA. 4 : 1 B. 1 : 2 C. 1 : 4 D. 2 : 159. The ray used for determining the crystal structure of solid isA. α -ray B. β -ray C. γ -ray D. X-ray60. For the structural analysis of crystals X-ray are used becauseA. X-rays have wavelength of the order of the inter-atomic spacingB. X-rays are highly penetrating radiationC. wavelength of X-rays is of order of nuclear sizeD. X-rays are coherent radiation61. The ratio of the molar amounts of H 2 S needed to precipitate the metal ions from 20 ml eachof 1 M Cd (NO 3 ) 2 and 0.5 M CuSO 4 isA. 2:1 B. 1:1 C. 1:2 D. indefinite62. Among the following elements, which one has the highest value of first ionization potential?A. Argon B. Barium C. Cesium D. Oxygen63. Which of the following concepts best explains that o-nitrophenol is more volatile than p-nitrophenol?A. Resonance B. Conjugation C. Hydrogen binding D. Covalent bonding64. Which of the following statements is false?A. Ionic compounds generally have low m.p.and b.p.B. Carbon tetrachloride is a non-polar molecule


C. Anhydrous AlCl 3 is a covalent substanceD. A molecule represents a more stable state as compared to individual atoms65. The chemical species having same number of electrons in the outermost and penultimateshell isA. Al 3+ B. O 2- C. Na + D. Cl -66. The solution was prepared by dissolving 0.0005 mol of Ba (OH) 2 in 100 ml of the solution. Ifthe base is assume to ionize completely, the pOH of the solution will beA. 10 B. 12 C. 2 D. unpredictable67. In which of the following neutralization willthe enthalpy of neutralization be the smallest?A. H 3 PO 4 B. NaOH C. NaOHwith NaOH and with HClD. HClwithNH 4 OHCH 3 OOH68. The pH of 10 -8 M NaOH will beA. 6.96 B. 7.04 C. 12.0 D. 869. Gas deviates from ideal gas nature because moleculesA. attract each other B. contain covalent bondC. show Brownian movement D. are colourless70. Among the following reactions, the fastest one isA. precipitation of silver chloride by mixing silver nitrate and sodium chloride solutionsB. burning of coalC. rusting of iron in moist airD. conversion of monoclinic sulphur to rhombic sulphur71. When 5.0 g of BaCl 2 is dissolved in water to have 10 6 g of solution. The concentration ofsolution isA. 5M B. 5gmL -1 C. 2.5 ppm D. 5 ppm72. The unit of electrochemical equivalent isA. coulomb/gram B. gm-ampere C. gm./coulomb D. gm-ampere -173. Adsorption increases whenA. temperature remains B. temperatureconstantincreasesC. temperature D. none of the abovedecreases74. The number of hours required for a current of 3.0 A to decompose electrically 18 g of waterisA. 12 hours B. 24 hours C. 6 hours D. 18 hours75. The number of electrons per second, which pass through a cross section of a copper wirecarrying 10 -16 A, isA. 16 x 10 -2 e/s B. 1.6 x 10 -3 C. 60 e/s D. 625 e/s


76. 20 ml of HCl having certain normality neutralizes exactly 1.0 g CaCO 3 . The normality ofacid isA. 0.1 N B. 1.0 N C. 0.5 N D. 0.01 N77. The alkali metal used in photoelectric cell isA. Cs B. Fr C. K D. Rb78. Calcium is extracted fromA. fused CaSO 4 B. fused Ca 3 (PO 4 ) 3 C. fused CaCl 2D. aqueous CaCl 2solution79. SbCl 3 upon hydrolysis yieldsA. Sb(OH) 3 B. SbO + C. Sb +3 D. None of the above80. Which of the following trioxides can exist asmonomer molecule?A. SO 3 ingaseousstateB. TeO 3 C. SeO 3 inall statesD. SO 3 insolid state81. Pure chlorine is obtainedA. by heating PtCl 4B. by heating a mixture of NaCl and MnO 2 with conc. H 2 SO 4C. by heating MnO 2 with HClD. by treating bleaching powder with HCl82. Which of the following gases is used in very low temperature thermometers?A. N 2 B. H 2 C. Ne D. He83. Number of nucleons in D 2 molecule isA. 4 B. 1 C. 2 D. 384. There is no s-s bond inA. S 2 O 72-B. S 2 O 32-C. S 2 O 42-D. S 2 O 52-85. The ratio of C p /C v for inert gas isA. 1.66 B. 1.33 C. 1.99 D. 2.1386. Electrolytic reduction method is used in theextraction ofA. highlyB. transition metalselectropositive elementsC. noble metals D. highlyelectronegativeelements87. The metal that is extracted from sea water isA. Mg B. Au C. Ca D. Fe


88. The compound having blue colour isA. HgSO 4 B. PbSO 4 C. CuSO 4 .5H 2 O D. CuSO 489. Which of the following is known as ‘Wol-framite’?A. Na 2 CO 3 + K 2 CO 3 B. FeWO 4 C. SnO 2 D. 98% pure Zinc90. Within each transition series, the oxidation stateA. first decreases till the middle of period and then increasesB. decreases regularly in moving from left to rightC. first increases till the middle of period and then decreasesD. none of the trend is correct91. Which of the following properties of graphite and diamond are identical?A. Density B. Crystal structure C. Atomic weightD. Electricalconductivity92. Which of the following is an example of copolymer?A. PAN B. PTFE C. D. Buna-SPolythene93. The reagent which forms crystalline osazone derivative when reacted with glucose isA. Hydroxylamine B. Benedict solution C. Fehling solution D. Phenylhydrazine94. To which class of dyes does phenolphthalein belong?B. Triphenyl methaneA. Phthalein dyesC. Nitro dyes D. Azo dyesdyes95. Peroxo linkage is present inA. H 2 S 2 O 8 B. H 2 SO 3 C. H 2 S 2 O 7 D. H 2 SO 496. Tautomerism is exhibited byA. RCH 2 NO 2 B. R 3 CNO 2 C. (CH 3 ) 2 NH D. (CH 3 ) 3 CNO97. Latest technique for purification, isolation and separation of organic substances isA. chromatography B. sublimation C. crystallization D. distillation98. Lactic acid looses optical activity when reduced with red P and HI becauseA. racemic mixture is formed B. spatial arrangement is changedC. symmetry of the molecule is destroyed D. chirality of the molecule is destroyed99. In order to convert aniline intochlorobenzene, the reagents needed areA.Cl 2 /AlCl 3C.B. Cl 2 /CCl 4 NaNO 2 /HCl D. CuCland CuCl100. Which of the following alcohol on dehydration with conc. H 2 SO 4 will yield 2-butene?A. 2-methyl-2-propanol B. 2-methyl-2-butanol C. 2-propanol D. Sec. Butyl alcohol


101. A compound A has a molecular formula C 2 Cl 3 OH. It reduces Fehling solution and anoxidation gives a monocarboxylic acid B. It can be obtained by the action of chlorine on ethylalcohol. A isD. MonochloroaceticA. Chloral B. Chloroform C. Methyl chlorideacid102. Which of the following will yield Benzaldimine hydrochloride?A. benzonitrile and SnCl 2 /HCl B. nitrobenzene and SnCl 2 /HClC. benzene and hydrazine D. hydrazine and HCl103. Isopropyl alcohol is heated on a water bath with the suspension of bleaching powder. Whichof the following products will be formed?A. Propene B. Ethanol C. Isopropyl chloride D. Trichloromethane104. Which of the following compounds is least basic?A. C 6 H 5 NH 2 B. C 2 H 5 NH 2 C. CH 3 NH 2 D. NH3105. Iodine dissolves in KI solution due to theformation ofA. I + B. I - --C. I 2 D. I 3106. Hydrogen sulphide exhibitsA. acidic properties B. basic properties C. oxidising properties D. none of the above107. White Phosphorus reacts with caustic soda. The products are pH 3 and NaH 2 PO 2 . Thisreaction is an example ofA. oxidation B. reduction C. oxidation and D. neutralisationreduction108. Ammonia solution dissolves fairly inA. Hg 2 Cl 2 B. PbCl 2 C. Cu(OH) 2 D. AgI109. Amongst the trihalides of nitrogen, which one is the least basic?A. NF 3 B. NCl 3 C. NBr 3 D. NI 3110. Among the various allotropes of carbon,A. diamond is the B. graphite is thehardesthardestC. lamp black is thehardest111. Bone charcoal is used for decolourising sugar because itA. reduces colouring matter B. oxidises colouring matterC. absorbs colouring matter D. none of the above112. Tin (II) chloride is used as aA. mordantin dyingB. catalystC.oxidisingagentD. none ofthe aboveD. coke is the hardest


113. Inert pair effect is most prominent inA. aluminium B. boron C. gallium D. thallium114. In the alumino thermite process, aluminium acts asA. an oxidising agent B. a flux C. a reducing agent D. a solder115. The correct structure of mercurous ion isA. Hg + B. Hg 2+ C. Hg 2+D. Hg 22+116. Which one of the following is purely ionic?A. Sodium chloride B. Beryllium chloride C. Lithium chloride D. Carbon tetrachloride117. A compound 'A' on heating gives a colourless gas. The residue is dissolved in water toobtain B. Excess CO 2 is passed through aqueous solution of B, when C is formed. C on gentleheating gives back A. The compound A isA. NaHCO 3 B. Na 2 CO 3 C. Ca(HCO 3 ) 2 D. CaCO 3118. A solution of sodium sulphate in water iselectrolysed using inert electrodes. The productsat the cathode and anode are respectivelyA. H 2 , O 2 B. O 2 , H 2 C. O 2 , Na D. O 2 , SO 2119. The metals occurring in the form of their compound in the earth's crust are calledA. matters B. minerals C. alloys D. gangue120. A commercial sample of hydrogen peroxide is labelled as 10 volume. Its percentagestrength is nearlyA. 1% B. 3% C. 10% D. 90%121. If (1 + x) n = P 0 + P 1 + P 2 x + P 2 x 2 + ...... + P n x n , then the value of P 0 - P 2 + P 4 - ....... isA. 2 n cosnπ/4 B. 2 n/2 cosnπ/4 C. 2 n/2 sinnπ/4 D. 2 n sinnπ/4122. If a, b, c and x are real numbers, then x 2 + 2bx + c will be positive ifA. b 2 > c B. b 2 < c C. b 2 > 4c D. b 2 < 4c123. The one of the values of (-i) 1/3 isA. (1/2)(√3 - i) B. (-1/2)(√3 + i) C. ± (1/2)(√3 + i) D. none of the above124. Let A = R ≈ {m}and B = R ≈ {n}, where R is a set of real numbers. Let f(x) = (x - n)/(x - m),then f is (where m, n are any integers)A. one-one onto B. many one onto C. one-one into D. many one into125. Cards are dealt one by one from a well shuffled pack until an ace appears. The probabilitythat exactly n cards are dealt with before the first ace appears is


A. [4(51 - n)(50 - n)(49 - n)]/(13.51.50.49) B. 4/(52 - n)C. [48 - (n - 1)]/(52 - n) D. none of the above126. A determinant is chosen at random fromthe set all determinants of order 2 with element0 and only. The probability that the value ofdeterminant chosen is positive, isA. 11/18 B. 11/14 C. 13/16 D. 3/16127. The value of theintegral| 1 - x | dx equalsA. 1 B. 2 C. 4 D. 0128. The domain of the function f(x) =sin -1 log 2 (x 2 /2) isA. [-2, 2] ≈ {0} B. [-1, 1] ≈ {0} C. [-2, 2] D. [-1, 1]129. Lt (1 - x) [(tanπx)/2] equalsx → 0A. π/2 B. 2/π C. π - 2 D. π + 2130. The function f(x) = | x |/x; x ≠ 0 and f(x) = 1; x = 0 is discontinuousatA. x = 0 B. x = 1 C. x = 2 D. x = -2131. If x = a (t - sint), y = a (t - cost), then d 2 y/dx 2 is equal toA. (1/4a)(cosec 2 t/2) B. (1/4a)(cosec 3 t/2) C. - [(1/4a)(cosec 2 t/3)] D. - [(1/4a)(cosec 4 t/2)]132. If x, y, and z are arithmetic, geometric, and harmonic means respectively of two distinctposition numbers, thenA. z < y < x B. x < y < z C. x < z < y D. x > z > y133. All the solutions of the equation 16xy + x 2 + y 2 - 8x - 8y - 20 = 0 representsA. a straight line B. pair of straight lines C. a circle D. a parabola134. The solution set of an inequality 5 - 15y > 125, y ∈ R isA. { y | y ∈ R } B. { y | y > 6 } C. { y | y < -8 } D. { y | y ∈ 8 & y ∈ 9 }135. Unit vector in the xy-plane that makes an angle of 45 o with the vector i + j and an angle of60 o with the vector 3i - 4j isA. i B. 2i C. √2i D. none of the above136. Given the line (x + 3)/2 = (y - 4)/3 = (z + 5)/2 and the plane 4x - 2y


- z = 1,then the line isA. perpendicular to theplaneC. inclined with 45 o tothe planeB. inclined with 60 o tothe planeD. parallel to the planeLt [x sinx + log (1 - x) x ]/x 3137.equalsx → 0A. 1/2 B. - 1/2 C. 1/4 D. - 1/4138. Four numbers are such that the first three are in A.P., while the last three are in G.P. Thefirst number is 6 and common ratio of G.P. is 1/2, then the numbers areA. 2, 4, 6, 8 B. 6, 4, 2, 1 C. 6, 4, 3, 2 D. 6, 9, 3, 1139. If the arithmetic and geometric mean of two distinct positive numbers are A and Grespectively, then their harmonic mean isA. A/√G B. A/G 2 C. G 2 /A D. √A/G140. The area bounded by the straight lines y = 1, x + y = 2, and x - y = 2 isA. 11 B. 11/2 C. 1/2 D. 2/11141. The value of 5 2 log 25 5 isA. 4 B. 5 C. 6 D. 8142. If the angle of intersection between the curves y = x 2 and y 2 = 4x, then the point ofintersection isA. (0, 0) B. (0, 1) C. (1, 0) D. (1, 1)143. The pair of points which lie on the same side of the straight line 3x - 8y = 7 isA. (-4, -3), (1, 1) B. (0, 1), (3, 0) C. (-1, -1), (3, -7) D. (-1, -1), (3, 7)144. The equation x 2 - 8x + 16 = 0 hasA. coincident root B. imaginary root C. unequal root D. none of the above145. If b = 3, c = 4 and B = π/4, then the number of triangles that can be formed isA. 1 B. 2 C. 3 D. none of the above146. Lim (tan mθ)/m equalsθ → 0A. θ B. - θ C. θ 2 D. 0147. The range of the function f(x)[1 - x] - 1 = 0 isA. a set of irrationalnumbersB. a set of rationalnumbers


C. a set of real numbers D. none of the above148. If a, b, c are in A.P., thenA. 1/(a - b) = 1/(b - c) B. (a - b)/(b - c) = 2 C. (a - c)/2 = b D. b + c = 2a149. The sum of all numbers greater than 1000 formed by using the digits 1, 3, 5, 7, no digitrepeated in any number isA. 106656 B. 101276 C. 82171 D. 81273150. The vertices of a triangle are represented by the complex numbers 4 - 2i, -1 + 4i, and 6 + i,then the complex number representing the centroid of a triangle isA. 3 + i B. 3 - i C. 9 + i D. 9 - i151. sin (π + θ) sin (π - θ) cosec 2 θ is equal toA. sin θ B. cos θ C. 1 D. -1152. In a triangle ABC, [(b 2 - c 2 )/a]cos A + [(c 2 - b 2 )/a]cos B + [(a 2 - b 2 )/a]cos C is equal toA. abc B. 1/abc C. a 2 b 2 c 2 D. 0153. If ex-radii r 1 , r 2 , r 3 of a triangle ABC are in H.P., then the sides ofthe triangle are inD. none ofA. A.P. B. G.P. C. H.P.the above154. The vertices of a triangle are A(6, 4), B(4, -3) and C(-2, 3), which one of the following istrue for triangle ABC?B. an equilateral C. a right angledA. an isosceles triangle D. none of the abovetriangletriangle155. The length of tangent from (5, 1) to the circle x 2 + y 2 - 6x + 4y + 3 = 0 isA. 7 B. 14 C. 28 D. 36156. If a = i + 2j + k 4i + 3j - 2k, then the projection of b on ab =andisA. 2/√29 B. 5/√29 C. 3/√29 D. 2157. Which one is true?A. P(A/B) = P(A) +P(AB)B. P(A/B) = P(A) -P(B)C. P(A/B) =[P(AB)]/P(B)D. P(A/B) = P(A) -P(B/A)158. If y = (1/2)[log (tanx)], then the value of dy/dx at x = π/4 isA. 1 B. 0 C. -1 D. ∞159. If y = (tanx + secx) x , then dy/dx is equal toA. x secx B. y secx C. m secx D. mxy


160. The equation 2x 2 + 3x + 1 = 0 hasA. rational root B. irrational root C. equal root D. none of the above161. A bag contains 6 red, 5 green, and 7 white balls. The probability of choosing a red or awhite ball isA. 1/3 B. 11/13 C. 13/18 D. 3/8162. ∫ (x + 2)/(x + 4) dx is equal toA. 1/2[tan -1 (x - 2/x)] +B. tan -1 x + ccC. 1/2[tan -1 (2/x)] + c D. none of the above163. The length intercepted on the line 3x + 4y + 1 = 0 by the circle (x - 1) 2 + (y - 4) 2 = 25 isA. 3 B. 4 C. 5 D. 6164. The period of the function cos [(3/5)α] - sin [(2/7)α] isA. 7π B. 10π C. 70π D. 3π165. The minimum value of x x is attained when x is equal toA. - e B. + e C. e 2 D. 1/e166. If a, b, c and u, v, w are complex numbers representing the verticesof two triangles such that c = (1 - r)a + rb and w = (1 - r)u + rv, where ris a complex number, then the two triangles areA. similarB.congruentC. equal inareaD. equalbases167. In a triangle ABC, if r and R are the in-radius and circum-radius respectively, then (a cos A+ b cos B + c cos C)/(a + b + c) isA. r/R B. R/r C. R 2 /r D. r 2 /R168. ∫ [(x + sinx)/(1 + cosx)] dx is equal toA. x tan(x/2) B. x tan(x/2) + c C. log (1 + cosx) + c D. x log (cos x) + c169. The differential coefficient of f [log(x)] when f(x) log x isA. x log x B. x/(log x) C. 1/(x log x) D. (log x)/x170. If x = 9 sin 2θ (1 + cos 2θ) and y = b cos 2θ (1 - cos 2θ), then the value of dy/dx isA. (b tan θ)/a B. a/(b tan θ) C. (a tan θ)/b D. ab tan θ171. The number of solution of the equation (tan x + sec x = 2 cos x) lying in the interval (0, 2π)isA. 0 B. 1 C. 2 D. 3172. If θ and φ are angles in the first quadrant such that tan θ = 1/7 andsin φ = 1/√10, then


A. θ + 2φ = B. θ + 2φ = C. θ + 2φ = D. θ + 2φ =90 o 60 o 30 o 45 o173. If a cos 2θ + b sin 2θ = c has a and b as its solution, then the value of tan α + tan β isA. (c + a)/2b B. 2b/(c + a) C. (c - a)/2b D. b/(c + a)174. The perimeter of a certain sector of a circle is equal to the length of the arc of a semi-circlehaving the same radius, the angle of the sector isA. 65 o 24' B. 64 o 24' C. 63 o 24' D. 62 o 24'175. The value of tan -1 x + cot -1 x isA. π/3 B. π/6 C. 2π/3 D. 2π176. If a circle cuts a rectangular hyperbola xy = c 2 in A, B, C, D and the parameters of thesefour points be t 1 , t 2 , t 3 and t 4 respectively, thenA. t 1 t 2 = t 3 t 4 B. t 1 t 2 t 3 t 4 = 1 C. t 1 = t 2 D. t 3 = t 4177. If the normal to y 2 = 12x at (3, 6) meets theparabola again in (27, -8) and the circle on thenormal chord as diameter isA. x 2 + y 2 + 30x + 12y - B. x 2 + y 2 + 30x + 12y27 = 0C. x 2 + y 2 - 30x - 12y -27 = 0+ 27 = 0D. x 2 + y 2 - 30x + 12y -27 = 0178. If the normal any point P on the ellipse cuts the major and the minor axes in G and grespectively and C be the centre of the ellipse, thenA. a 2 (CG) 2 + b 2 (Cg) 2 = (a 2 - b 2 ) 2 B. a 2 (CG) 2 - b 2 (Cg) 2 = (a 2 - b 2 ) 2C. a 2 (CG) 2 - b 2 (Cg) 2 = (a 2 + b 2 ) 2 D. none of the above179. The point of intersection of the tangent at the end of the latus rectum of the parabola y 2 = 4xisA. (-1, 1) B. (1, 1) C. (-1, 0) D. (0, 0)180. If a, b, c are distinct positive numbers, then the expression (b + c - a)(c + a - b)(a + b - c) -abc isA. positive B. negativeC. both negative and positive D. none of the above

Hooray! Your file is uploaded and ready to be published.

Saved successfully!

Ooh no, something went wrong!